+ All Categories
Home > Documents > INTERMEDIATE 2 – ADDITIONAL QUESTION BANK UNIT 3 : Further Trig Algebraic Operations Quadratic...

INTERMEDIATE 2 – ADDITIONAL QUESTION BANK UNIT 3 : Further Trig Algebraic Operations Quadratic...

Date post: 31-Dec-2015
Category:
Upload: avis-hoover
View: 226 times
Download: 3 times
Share this document with a friend
216
INTERMEDIATE 2 – ADDITIONAL QUESTION BANK UNIT 3 : Further Trig Algebraic Operations Quadratic Functions EXIT
Transcript
Page 1: INTERMEDIATE 2 – ADDITIONAL QUESTION BANK UNIT 3 : Further Trig Algebraic Operations Quadratic Functions EXIT.

INTERMEDIATE 2 – ADDITIONAL QUESTION BANK

UNIT 3 :

FurtherTrig

AlgebraicOperations

Quadratic Functions

EXIT

Page 2: INTERMEDIATE 2 – ADDITIONAL QUESTION BANK UNIT 3 : Further Trig Algebraic Operations Quadratic Functions EXIT.

INTERMEDIATE 2 – ADDITIONAL QUESTION BANK

UNIT 3 : AlgebraicOperations

You have chosen to study:

Please choose a question to attempt from the following:

1 2 3 4

EXITBack to

Unit 3 Menu

5 6 7 8

Page 3: INTERMEDIATE 2 – ADDITIONAL QUESTION BANK UNIT 3 : Further Trig Algebraic Operations Quadratic Functions EXIT.

Express (m -7)

as a single fraction in its simplest form.

ALGEBRAIC OPERATIONS : Question 1

Go to full solution

Go to Comments

Go to Algebraic Ops Menu

Reveal answer only

EXIT

Get hint

4

2 ( 7)

m m

m

What would you like to do now?

Page 4: INTERMEDIATE 2 – ADDITIONAL QUESTION BANK UNIT 3 : Further Trig Algebraic Operations Quadratic Functions EXIT.

Express (m -7)

as a single fraction in its simplest form.

ALGEBRAIC OPERATIONS : Question 1

EXIT

4

2 ( 7)

m m

m

Use the pattern

ab + c

dad + bc

bd=

Go to full solution

Go to Comments

Go to Algebraic Ops Menu

Reveal answer only

What would you like to do now?

Page 5: INTERMEDIATE 2 – ADDITIONAL QUESTION BANK UNIT 3 : Further Trig Algebraic Operations Quadratic Functions EXIT.

Express (m -7)

as a single fraction in its simplest form.

ALGEBRAIC OPERATIONS : Question 1

EXIT

4

2 ( 7)

m m

m

m2 - m

2m + 14=

Go to full solution

Go to Comments

Go to Algebraic Ops Menu

Try another like this

What would you like to do now?

Page 6: INTERMEDIATE 2 – ADDITIONAL QUESTION BANK UNIT 3 : Further Trig Algebraic Operations Quadratic Functions EXIT.

Comments

Begin Solution

Question 1

Menu

Back to Home

Express

(m -7)

as a single fraction in its

simplest form.

4

2 ( 7)

m m

m

1. Use the pattern

ab + c

dad + bc

bd=

4

2 ( 7)

m m

m

= m (m + 7) - (2 x 4m)

2 (m + 7)

m2 + 7m - 8m2m + 14

=

m2 - m2m + 14

=Try another like this

Page 7: INTERMEDIATE 2 – ADDITIONAL QUESTION BANK UNIT 3 : Further Trig Algebraic Operations Quadratic Functions EXIT.

Comments

Menu

Back to Home

1. Use the pattern

ab + c

dad + bc

bd=

4

2 ( 7)

m m

m

m (m + 7) - (2 x 4m)

2 (m + 7)

m2 + 7m - 8m2m + 14

=

m2 - m2m + 14

=

To add or subtract fractions

use the results:

ab

+cd

ad + bcbd

=

ab

-cd

ad - bcbd

=

Try another

Page 8: INTERMEDIATE 2 – ADDITIONAL QUESTION BANK UNIT 3 : Further Trig Algebraic Operations Quadratic Functions EXIT.

ALGEBRAIC OPERATIONS: Question 1B

Express (t 3)

as a single fraction in its simplest form.

7 4

10 ( 3)

t t

t

Go to full solution

Go to Comments

Go to Algebraic Ops Menu

Reveal answer only

EXIT

Get hint

What would you like to do now?

Page 9: INTERMEDIATE 2 – ADDITIONAL QUESTION BANK UNIT 3 : Further Trig Algebraic Operations Quadratic Functions EXIT.

ALGEBRAIC OPERATIONS: Question 1B

Express (t 3)

as a single fraction in its simplest form.

7 4

10 ( 3)

t t

t

Use the pattern

ab + c

dad + bc

bd=

Go to full solution

Go to Comments

Go to Algebraic Ops Menu

Reveal answer only

EXIT

What would you like to do now?

Page 10: INTERMEDIATE 2 – ADDITIONAL QUESTION BANK UNIT 3 : Further Trig Algebraic Operations Quadratic Functions EXIT.

ALGEBRAIC OPERATIONS: Question 1B

Express (t 3)

as a single fraction in its simplest form.

7 4

10 ( 3)

t t

t

7t2 + 19t

10t – 30 =

Go to full solution

Go to Comments

Go to Algebraic Ops Menu

EXIT

What would you like to do now?

Page 11: INTERMEDIATE 2 – ADDITIONAL QUESTION BANK UNIT 3 : Further Trig Algebraic Operations Quadratic Functions EXIT.

Question 1B

Express

(t 3)

as a single fraction in its

simplest form.

1. Use the pattern

ab + c

dad + bc

bd=

7 4

10 ( 3)

t t

t

= 7t (t – 3) + (10 x 4t)

10 (t – 3)

7t2 – 21t +40t10t – 30

=

7t2 + 19t10t – 30

=

7 4

10 ( 3)

t t

t

Comments

Begin Solution

Continue Solution

Menu

Back to Home

What would you like to do now?

Page 12: INTERMEDIATE 2 – ADDITIONAL QUESTION BANK UNIT 3 : Further Trig Algebraic Operations Quadratic Functions EXIT.

Comments

To add or subtract fractions

use the results:

ab

+cd

ad + bcbd

=

ab

-cd

ad - bcbd

=

1. Use the pattern

ab + c

dad + bc

bd=

7 4

10 ( 3)

t t

t

= 7t (t – 3) + (10 x 4t)

10 (t – 3)

7t2 – 21t +40t10t – 30

=

7t2 + 19t10t – 30

=Menu

Back to Home

Next Comment

Page 13: INTERMEDIATE 2 – ADDITIONAL QUESTION BANK UNIT 3 : Further Trig Algebraic Operations Quadratic Functions EXIT.

Comments

7t2 + 19t10t - 30

Note:

Always check that you have

cancelled as far as possible.

This is the final result,

it does not cancel further.

1. Use the pattern

ab + c

dad + bc

bd=

7 4

10 ( 3)

t t

t

= 7t (t – 3) + (10 x 4t)

10 (t – 3)

7t2 – 21t +40t10t – 30

=

7t2 + 19t10t – 30

=Menu

Back to Home

Page 14: INTERMEDIATE 2 – ADDITIONAL QUESTION BANK UNIT 3 : Further Trig Algebraic Operations Quadratic Functions EXIT.

ALGEBRAIC OPERATIONS: Question 2

2 2

3

2 15

5 4

a b

b a Express

as a single fraction in its simplest form.

Go to full solution

Go to Comments

Go to Algebraic Ops Menu

Reveal answer only

EXIT

Get hint

What would you like to do now?

Page 15: INTERMEDIATE 2 – ADDITIONAL QUESTION BANK UNIT 3 : Further Trig Algebraic Operations Quadratic Functions EXIT.

ALGEBRAIC OPERATIONS: Question 2

2 2

3

2 15

5 4

a b

b a Express

as a single fraction in its simplest form.

Go to full solution

Go to Comments

Go to Algebraic Ops Menu

Reveal answer only

EXIT

What would you like to do now?

Multiply top line then

bottom line.

Cancel numbers

then letters in alphabetical

order.

Page 16: INTERMEDIATE 2 – ADDITIONAL QUESTION BANK UNIT 3 : Further Trig Algebraic Operations Quadratic Functions EXIT.

ALGEBRAIC OPERATIONS: Question 2

2 2

3

2 15

5 4

a b

b a Express

as a single fraction in its simplest form.

Go to full solution

Go to Comments

Go to Algebraic Ops Menu

EXIT

Try another like this

What would you like to do now?

3b2a=

Page 17: INTERMEDIATE 2 – ADDITIONAL QUESTION BANK UNIT 3 : Further Trig Algebraic Operations Quadratic Functions EXIT.

Question 2

Express

as a single fraction in its

simplest form.

2 2

3

2 15

5 4

a b

b a

2a2

5b x 15b2

4a3

30a2b2

20a3b=

1. Multiply top line then bottom line.

2. Cancel numbers then letters in alphabetical order.

3

2 a

b

Comments

Begin Solution

Menu

Back to Home

Try another like this

3b2a=

What would you like to do now?

Page 18: INTERMEDIATE 2 – ADDITIONAL QUESTION BANK UNIT 3 : Further Trig Algebraic Operations Quadratic Functions EXIT.

Comments

2a2

5b x 15b2

4a3

30a2b2

20a3b=

3b2a=

1. Multiply top line then bottom line.

2. Cancel numbers then letters in alphabetical order.

3

2 a

b

To multiply fractions use the result:

ab x c

dacbd=

Menu

Back to Home

Next Comment

Page 19: INTERMEDIATE 2 – ADDITIONAL QUESTION BANK UNIT 3 : Further Trig Algebraic Operations Quadratic Functions EXIT.

Comments

2a2

5b x 15b2

4a3

30a2b2

20a3b=

3b2a=

1. Multiply top line then bottom line.

2. Cancel numbers then letters in alphabetical order.

3

2 a

b

To simplify final answer write

out in full and cancel:

30a2b2

20a3b=

= 3b2a

30.a.a.b.b20.a.a.a.b

Menu

Back to Home

Try another

Page 20: INTERMEDIATE 2 – ADDITIONAL QUESTION BANK UNIT 3 : Further Trig Algebraic Operations Quadratic Functions EXIT.

ALGEBRAIC OPERATIONS: Question 2B

2

3

2 6v v

w w Express

as a single fraction in its simplest form.

Go to full solution

Go to Comments

Go to Algebraic Ops Menu

Reveal answer only

EXIT

Get hint

What would you like to do now?

Page 21: INTERMEDIATE 2 – ADDITIONAL QUESTION BANK UNIT 3 : Further Trig Algebraic Operations Quadratic Functions EXIT.

ALGEBRAIC OPERATIONS: Question 2B

Express

as a single fraction in its simplest form.

Multiply top line then

bottom line.

Cancel numbers

then letters in alphabetical

order.

2

3

2 6v v

w w

To divide by a fraction :

turn it upside down and multiply.

Go to full solution

Go to Comments

Go to Algebraic Ops Menu

Reveal answer only

EXIT

What would you like to do now?

Page 22: INTERMEDIATE 2 – ADDITIONAL QUESTION BANK UNIT 3 : Further Trig Algebraic Operations Quadratic Functions EXIT.

ALGEBRAIC OPERATIONS: Question 2B

Express

as a single fraction in its simplest form.

2

3

2 6v v

w w vw2

3=

Go to full solution

Go to Comments

Go to Algebraic Ops Menu

EXIT

What would you like to do now?

Page 23: INTERMEDIATE 2 – ADDITIONAL QUESTION BANK UNIT 3 : Further Trig Algebraic Operations Quadratic Functions EXIT.

Question 2B

Express

as a single fraction in its

simplest form.

1. To divide by a fraction : turn it upside down and multiply.

3. Cancel numbers then letters in alphabetical order.

1

3

w2v

2

3

2 6v v

w w

2v2

w 6v w3

2v2

w x 6vw3

=

2. Multiply top line then bottom line.

2v2w3

6vw=

vw2

3=Comments

Begin Solution

Continue Solution

Menu

Back to HomeWhat would you like to do now?

Page 24: INTERMEDIATE 2 – ADDITIONAL QUESTION BANK UNIT 3 : Further Trig Algebraic Operations Quadratic Functions EXIT.

Comments

To divide fractions use the result:1. To divide by a fraction : turn it

upside down and multiply.

3. Cancel numbers then letters in alphabetical order.

2v2

w 6v w3

2v2

w x 6vw3

=

2. Multiply top line then bottom line.

2v2w3

6vw=

vw2

3=

1

3

w2v

ab ÷ c

dadbc= a

b x dc =

Menu

Back to Home

Next Comment

Page 25: INTERMEDIATE 2 – ADDITIONAL QUESTION BANK UNIT 3 : Further Trig Algebraic Operations Quadratic Functions EXIT.

Comments

1. To divide by a fraction : turn it upside down and multiply.

3. Cancel numbers then letters in alphabetical order.

2v2

w 6v w3

2v2

w x 6vw3

=

2. Multiply top line then bottom line.

2v2w3

6vw=

vw2

3=

1

3

w2v

To simplify final answer write

out in full and cancel:

2v2w3

6vw= 2.v.v.w.w.w

6.v.w

vw2

3=

Menu

Back to Home

Next Comment

Page 26: INTERMEDIATE 2 – ADDITIONAL QUESTION BANK UNIT 3 : Further Trig Algebraic Operations Quadratic Functions EXIT.

ALGEBRAIC OPERATIONS: Question 3 5 3

4 424 8d d

Simplify

Go to full solution

Go to Comments

Go to Algebraic Ops Menu

Reveal answer only

EXIT

Get hint

What would you like to do now?

Page 27: INTERMEDIATE 2 – ADDITIONAL QUESTION BANK UNIT 3 : Further Trig Algebraic Operations Quadratic Functions EXIT.

Deal with numbers and then apply laws

of indices: when dividing subtract

the powers.

Remember subtracting negative is like adding.

ALGEBRAIC OPERATIONS: Question 3

Simplify 5 3

4 424 8d d

Go to full solution

Go to Comments

Go to Algebraic Ops Menu

Reveal answer only

EXIT

What would you like to do now?

Page 28: INTERMEDIATE 2 – ADDITIONAL QUESTION BANK UNIT 3 : Further Trig Algebraic Operations Quadratic Functions EXIT.

ALGEBRAIC OPERATIONS: Question 3

Simplify 5 3

4 424 8d d

= 3d2

Go to full solution

Go to Comments

Go to Algebraic Ops Menu

EXIT

What would you like to do now?

Try another like this

Page 29: INTERMEDIATE 2 – ADDITIONAL QUESTION BANK UNIT 3 : Further Trig Algebraic Operations Quadratic Functions EXIT.

Simplify

.

Question 3 1. Deal with numbers and then apply

laws of indices: when dividing subtract the powers.

2. Remember subtracting negative is like adding.

5 3

4 424 8d d

24d5/4 8d–3/4

= 3d5/4-(–3/4)

= 3d8/4

= 3d2

Comments

Begin Solution

Menu

Back to Home

Try another like this

Page 30: INTERMEDIATE 2 – ADDITIONAL QUESTION BANK UNIT 3 : Further Trig Algebraic Operations Quadratic Functions EXIT.

Comments

1. Deal with numbers and then apply laws of indices: when dividing subtract the powers.

2. Remember subtracting negative is like adding.

24d5/4 8d–3/4

= 3d5/4-(–3/4)

= 3d8/4

= 3d2

Learn Laws of Indices:

m n m na a a

5 23 3a a

e.g.

73a

5 2( )3 3a

5 23 3a

Menu

Back to Home

Try another

Page 31: INTERMEDIATE 2 – ADDITIONAL QUESTION BANK UNIT 3 : Further Trig Algebraic Operations Quadratic Functions EXIT.

ALGEBRAIC OPERATIONS: Question 3B 5 2

3 3

2

a a

a

Simplify

Go to full solution

Go to Comments

Go to Algebraic Ops Menu

Reveal answer only

EXIT

Get hint

What would you like to do now?

Page 32: INTERMEDIATE 2 – ADDITIONAL QUESTION BANK UNIT 3 : Further Trig Algebraic Operations Quadratic Functions EXIT.

Deal with top row first. Apply laws of indices: when dividing subtract the

powers & when multiplying add

powers.

Now divide remembering subtracting negative is like adding.

ALGEBRAIC OPERATIONS: Question 3B

Simplify

5 2

3 3

2

a a

a

Go to full solution

Go to Comments

Go to Algebraic Ops Menu

Reveal answer only

EXIT

What would you like to do now?

Page 33: INTERMEDIATE 2 – ADDITIONAL QUESTION BANK UNIT 3 : Further Trig Algebraic Operations Quadratic Functions EXIT.

ALGEBRAIC OPERATIONS: Question 3B

Simplify

5 2

3 3

2

a a

a

= a3

Go to full solution

Go to Comments

Go to Algebraic Ops Menu

EXIT

What would you like to do now?

Page 34: INTERMEDIATE 2 – ADDITIONAL QUESTION BANK UNIT 3 : Further Trig Algebraic Operations Quadratic Functions EXIT.

Simplify

.

Question 3B 1. Deal with top row first. Apply laws

of indices: when dividing subtract the powers & when multiplying add powers.

2. Now divide remembering subtracting negative is like adding.

5 2

3 3

2

a a

a

a-2

a5/3 x a–2/3

= a1-(-2)

= a5/3 –2/3 a-2

= a3/3

a-2

= a3 Comments

Begin Solution

Continue Solution

Menu

Back to Home

Page 35: INTERMEDIATE 2 – ADDITIONAL QUESTION BANK UNIT 3 : Further Trig Algebraic Operations Quadratic Functions EXIT.

Comments

Learn Laws of Indices:

m n m na a a 1. Deal with top row first. Apply laws

of indices: when dividing subtract the powers & when multiplying add powers.

2. Now divide remembering subtracting negative is like adding.

a-2

a5/3 x a–2/3

= a1-(-2)

= a5/3 –2/3 a-2

= a3/3

a-2

= a3

m n m na a a 5 2

3 3a a

e.g.

5 2( )3 3a

33a

1a

Menu

Back to Home

Next Comment

Page 36: INTERMEDIATE 2 – ADDITIONAL QUESTION BANK UNIT 3 : Further Trig Algebraic Operations Quadratic Functions EXIT.

Comments

Learn Laws of Indices:

m n m na a a 1. Deal with top row first. Apply laws

of indices: when dividing subtract the powers & when multiplying add powers.

2. Now divide remembering subtracting negative is like adding.

a-2

a5/3 x a–2/3

= a1-(-2)

= a5/3 –2/3 a-2

= a3/3

a-2

= a3

m n m na a a e.g.

2

a

a

1 ( 2)a 1 2a 3a

Menu

Back to Home

Next Comment

Page 37: INTERMEDIATE 2 – ADDITIONAL QUESTION BANK UNIT 3 : Further Trig Algebraic Operations Quadratic Functions EXIT.

ALGEBRAIC OPERATIONS: Question 4

1 5 4

3 3 3m m m

Simplify giving your

answer with positive indices.

Go to full solution

Go to Comments

Go to Algebraic Ops Menu

Reveal answer only

EXIT

Get hint

What would you like to do now?

Page 38: INTERMEDIATE 2 – ADDITIONAL QUESTION BANK UNIT 3 : Further Trig Algebraic Operations Quadratic Functions EXIT.

ALGEBRAIC OPERATIONS: Question 4

1 5 4

3 3 3m m m

Simplify giving your

answer with positive indices.

Deal with numbers and then apply laws

of indices: when dividing subtract

the powers.

Remember subtracting negative is like adding.

Go to full solution

Go to Comments

Go to Algebraic Ops Menu

Reveal answer only

EXIT

What would you like to do now?

Page 39: INTERMEDIATE 2 – ADDITIONAL QUESTION BANK UNIT 3 : Further Trig Algebraic Operations Quadratic Functions EXIT.

ALGEBRAIC OPERATIONS: Question 4

1 5 4

3 3 3m m m

Simplify giving your

answer with positive indices. = m2 + 1

m

Go to full solution

Go to Comments

Go to Algebraic Ops MenuEXIT

What would you like to do now?

Try another like this

Page 40: INTERMEDIATE 2 – ADDITIONAL QUESTION BANK UNIT 3 : Further Trig Algebraic Operations Quadratic Functions EXIT.

Question 4 1. Multiply out brackets remembering

to apply laws of indices: when multiplying add the powers.

2. Negative powers become positive on bottom line.

1 5 4

3 3 3m m m

Simplify

giving your answer with

positive indices.

m1/3( m5/3 + m-4/3 )

= m6/3 + m-3/3

= m2 + m-1

= m2 + 1 m

Comments

Begin Solution

Menu

Back to Home

Try another like thisWhat would you like to do now?

Page 41: INTERMEDIATE 2 – ADDITIONAL QUESTION BANK UNIT 3 : Further Trig Algebraic Operations Quadratic Functions EXIT.

Comments

Learn Laws of Indices:

m n m na a a

e.g.

2

1

a

2a

1. Multiply out brackets remembering to apply laws of indices: when multiplying add the powers.

2. Negative powers become positive on bottom line.

m1/3( m5/3 + m-4/3 )

= m6/3 + m-3/3

= m2 + m-1

= m2 + 1 m

1mm

aa

Menu

Back to Home

Try another

Page 42: INTERMEDIATE 2 – ADDITIONAL QUESTION BANK UNIT 3 : Further Trig Algebraic Operations Quadratic Functions EXIT.

ALGEBRAIC OPERATIONS: Question 4B

1 1 3

4 4 4w w w

Simplify

giving your answer without indices.

Go to full solution

Go to Comments

Go to Algebraic Ops Menu

Reveal answer only

EXIT

Get hint

What would you like to do now?

Page 43: INTERMEDIATE 2 – ADDITIONAL QUESTION BANK UNIT 3 : Further Trig Algebraic Operations Quadratic Functions EXIT.

Simplify

giving your answer without indices.

Multiply out

brackets remembering to

apply laws of indices: when

multiplying add the powers.

Zero power is 1 and ½ power is

square root.

ALGEBRAIC OPERATIONS: Question 4B 1 1 3

4 4 4w w w

Go to full solution

Go to Comments

Go to Algebraic Ops Menu

Reveal answer only

EXIT

What would you like to do now?

Page 44: INTERMEDIATE 2 – ADDITIONAL QUESTION BANK UNIT 3 : Further Trig Algebraic Operations Quadratic Functions EXIT.

ALGEBRAIC OPERATIONS: Question 4B

Simplify

giving your answer without indices.

1 1 3

4 4 4w w w

= 1 - w

Go to full solution

Go to Comments

Go to Algebraic Ops MenuEXIT

What would you like to do now?

Page 45: INTERMEDIATE 2 – ADDITIONAL QUESTION BANK UNIT 3 : Further Trig Algebraic Operations Quadratic Functions EXIT.

Question 4B 1. Multiply out brackets remembering

to apply laws of indices: when multiplying add the powers.

2. Zero power is 1 and ½ power is square root.

Simplify

giving your answer without indices.

1 1 3

4 4 4w w w

w-1/4( w1/4 - w3/4 )

= w0 - w1/2

= 1 - w

= w-1/4+1/4 - w-1/4+3/4

Comments

Begin Solution

Continue Solution

Menu

Back to Home

Page 46: INTERMEDIATE 2 – ADDITIONAL QUESTION BANK UNIT 3 : Further Trig Algebraic Operations Quadratic Functions EXIT.

Comments

Learn Laws of Indices:

e.g.

1. Multiply out brackets remembering to apply laws of indices: when multiplying add the powers.

2. Zero power is 1 and ½ power is square root.

w-1/4( w1/4 - w3/4 )

= w0 - w1/2

= 1 - w

= w-1/4+1/4 - w-1/4+3/4

0

1

1

nn

a

a a

12

133

a a

a a

Menu

Back to Home

Next Comment

Page 47: INTERMEDIATE 2 – ADDITIONAL QUESTION BANK UNIT 3 : Further Trig Algebraic Operations Quadratic Functions EXIT.

ALGEBRAIC OPERATIONS: Question 5

Evaluate 7c3/4 when c = 16

What would you like to do now?

Go to full solution

Go to Comments

Go to Algebraic Ops Menu

Reveal answer only

EXIT

Get hint

Page 48: INTERMEDIATE 2 – ADDITIONAL QUESTION BANK UNIT 3 : Further Trig Algebraic Operations Quadratic Functions EXIT.

Deal with indices first. Power ¾ is 4th root cubed.

Evaluate root before

power

ALGEBRAIC OPERATIONS: Question 5

Evaluate 7c3/4 when c = 16

What would you like to do now?

Go to full solution

Go to Comments

Go to Algebraic Ops Menu

Reveal answer only

EXIT

Page 49: INTERMEDIATE 2 – ADDITIONAL QUESTION BANK UNIT 3 : Further Trig Algebraic Operations Quadratic Functions EXIT.

ALGEBRAIC OPERATIONS: Question 5

Evaluate 7c3/4 when c = 16 = 56

What would you like to do now?

Try another like this

Go to full solution

Go to Comments

Go to Algebraic Ops MenuEXIT

Page 50: INTERMEDIATE 2 – ADDITIONAL QUESTION BANK UNIT 3 : Further Trig Algebraic Operations Quadratic Functions EXIT.

Question 5 1. Deal with indices first. Power ¾ is

4th root cubed.

= 56

Evaluate

7c3/4 when c = 16 c3/4 = (4c)3

= (416)3

= (2)3

= 8

So 7c3/4 = 7 x 8

Comments

Begin Solution

Menu

Back to Home

Try another like this

What would you like to do now?

Page 51: INTERMEDIATE 2 – ADDITIONAL QUESTION BANK UNIT 3 : Further Trig Algebraic Operations Quadratic Functions EXIT.

Comments

Learn Laws of Indices:

1. Deal with indices first. Power ¾ is 4th root cubed.

= 56

= (4c)3

= (416)3

= (2)3

= 8

So 7c3/4 = 7 x 8

m n mna a

Think “Flower Power”:

Power on top

Root at bottom

Menu

Back to Home

Next Comment

Page 52: INTERMEDIATE 2 – ADDITIONAL QUESTION BANK UNIT 3 : Further Trig Algebraic Operations Quadratic Functions EXIT.

Comments

Learn Laws of Indices:

1. Deal with indices first. Power ¾ is 4th root cubed.

= 56

= (4c)3

= (416)3

= (2)3

= 8

So 7c3/4 = 7 x 8

m n mna ae.g. 4 3 43

4 3 4 433

4

8 8 ( 8)

2 16

a a

Always find root before power

Menu

Back to Home

Try another

Page 53: INTERMEDIATE 2 – ADDITIONAL QUESTION BANK UNIT 3 : Further Trig Algebraic Operations Quadratic Functions EXIT.

ALGEBRAIC OPERATIONS: Question 5B

Evaluate 10f -1/2 when f = 25

What would you like to do now?

Go to full solution

Go to Comments

Go to Algebraic Ops Menu

Reveal answer only

EXIT

Get hint

Page 54: INTERMEDIATE 2 – ADDITIONAL QUESTION BANK UNIT 3 : Further Trig Algebraic Operations Quadratic Functions EXIT.

ALGEBRAIC OPERATIONS: Question 5B

Evaluate 10f -1/2 when f = 25

Deal with indices first. Power – ½ is square root on

bottom line.

What would you like to do now?

Go to full solution

Go to Comments

Go to Algebraic Ops Menu

Reveal answer only

EXIT

Page 55: INTERMEDIATE 2 – ADDITIONAL QUESTION BANK UNIT 3 : Further Trig Algebraic Operations Quadratic Functions EXIT.

ALGEBRAIC OPERATIONS: Question 5B

= 2 Evaluate 10f -1/2 when f = 25

What would you like to do now?

Go to full solution

Go to Comments

Go to Algebraic Ops MenuEXIT

Page 56: INTERMEDIATE 2 – ADDITIONAL QUESTION BANK UNIT 3 : Further Trig Algebraic Operations Quadratic Functions EXIT.

Question 5B

Evaluate

10f -1/2 when f = 25

1. Deal with indices first. Power – ½ is square root on bottom line.

10f -1/2

= 10 f

= 1025

= 10 5

= 2

Comments

Begin Solution

Continue Solution

Menu

Back to Home

What would you like to do now?

Page 57: INTERMEDIATE 2 – ADDITIONAL QUESTION BANK UNIT 3 : Further Trig Algebraic Operations Quadratic Functions EXIT.

Comments

Learn Laws of Indices:

e.g.

1. Deal with indices first. Power – ½ is square root on bottom line.

10f -1/2

= 10 f

= 1025

= 10 5

= 2

1

1mm

nn

aa

a a

33

12

1 12

2 8

9 9 3

Menu

Back to Home

Next Comment

Page 58: INTERMEDIATE 2 – ADDITIONAL QUESTION BANK UNIT 3 : Further Trig Algebraic Operations Quadratic Functions EXIT.

ALGEBRAIC OPERATIONS: Question 6

f(x) = 7x1/2 . Find the value of x if f(x) = 63.

What would you like to do now?

Go to full solution

Go to Comments

Go to Algebraic Ops Menu

Reveal answer only

EXIT

Get hint

What would you like to do now?

Page 59: INTERMEDIATE 2 – ADDITIONAL QUESTION BANK UNIT 3 : Further Trig Algebraic Operations Quadratic Functions EXIT.

Equate both things that are equal to

f(x).

To get rid of square roots square both

sides.

ALGEBRAIC OPERATIONS: Question 6

f(x) = 7x1/2 . Find the value of x if f(x) = 63.

What would you like to do now?

Go to full solution

Go to Comments

Go to Algebraic Ops Menu

Reveal answer only

EXIT

Page 60: INTERMEDIATE 2 – ADDITIONAL QUESTION BANK UNIT 3 : Further Trig Algebraic Operations Quadratic Functions EXIT.

ALGEBRAIC OPERATIONS: Question 6

X = 81

f(x) = 7x1/2 . Find the value of x if f(x) = 63.

What would you like to do now?

Try another like this

Go to full solution

Go to Comments

Go to Algebraic Ops MenuEXIT

Page 61: INTERMEDIATE 2 – ADDITIONAL QUESTION BANK UNIT 3 : Further Trig Algebraic Operations Quadratic Functions EXIT.

Question 6 1. Equate both things that are equal to

f(x). f(x) = 7x1/2 .

Find the value of x

if f(x) = 63.

7x1/2 = 63

so x1/2 = 9

(7)

2. Remember power ½ is the square root.

so x = 9

3. Now square each side.

so x = 92

ie x = 81Comments

Begin Solution

Menu

Back to Home

Try another like this

What would you like to do now?

Page 62: INTERMEDIATE 2 – ADDITIONAL QUESTION BANK UNIT 3 : Further Trig Algebraic Operations Quadratic Functions EXIT.

Comments

Learn Laws of Indices:1. Equate both things that are equal

to f(x).

7x1/2 = 63

so x1/2 = 9

(7)

2. Remember power ½ is the square root.

so x = 9

3. Now square each side.

so x = 92

ie x = 81

12a a

Note:In solving the equation

Square both sides2

4

4

16

a

a

Menu

Back to Home

Try another

Page 63: INTERMEDIATE 2 – ADDITIONAL QUESTION BANK UNIT 3 : Further Trig Algebraic Operations Quadratic Functions EXIT.

ALGEBRAIC OPERATIONS: Question 6B

f(x) = 2x1/3 . Find the value of x if f(x) = 20.

What would you like to do now?

Go to full solution

Go to Comments

Go to Algebraic Ops Menu

Reveal answer only

EXIT

Get hint

Page 64: INTERMEDIATE 2 – ADDITIONAL QUESTION BANK UNIT 3 : Further Trig Algebraic Operations Quadratic Functions EXIT.

Equate both things that are equal to

f(x).

To get rid of cube roots cube both

sides.

ALGEBRAIC OPERATIONS: Question 6B

f(x) = 2x1/3 . Find the value of x if f(x) = 20.

What would you like to do now?

Go to full solution

Go to Comments

Go to Algebraic Ops Menu

Reveal answer only

EXIT

Page 65: INTERMEDIATE 2 – ADDITIONAL QUESTION BANK UNIT 3 : Further Trig Algebraic Operations Quadratic Functions EXIT.

ALGEBRAIC OPERATIONS: Question 6B

X = 1000

f(x) = 2x1/3 . Find the value of x if f(x) = 20.

What would you like to do now?

Go to full solution

Go to Comments

Go to Algebraic Ops MenuEXIT

Page 66: INTERMEDIATE 2 – ADDITIONAL QUESTION BANK UNIT 3 : Further Trig Algebraic Operations Quadratic Functions EXIT.

Question 6B 1. Equate both things that are equal to

f(x). f(x) = 2x1/3 .

Find the value of x

if f(x) = 20. 2. Remember power 1/3 is the cube

root.

3. Now cube each side.

ie x = 1000

2x1/3 = 20

so x1/3 = 10

(2)

so 3x = 10

so x = 103

Comments

Begin Solution

Continue Solution

Menu

Back to Home

What would you like to do now?

Page 67: INTERMEDIATE 2 – ADDITIONAL QUESTION BANK UNIT 3 : Further Trig Algebraic Operations Quadratic Functions EXIT.

Comments

Learn Laws of Indices:

133a a

Note:In solving the equation

cube both sides

3 2a 32 8a

1. Equate both things that are equal to f(x).

2. Remember power 1/3 is the cube root.

3. Now cube each side.

ie x = 1000

2x1/3 = 20

so x1/3 = 10

(2)

so 3x = 10

so x = 103

Menu

Back to Home

Next Comment

Page 68: INTERMEDIATE 2 – ADDITIONAL QUESTION BANK UNIT 3 : Further Trig Algebraic Operations Quadratic Functions EXIT.

ALGEBRAIC OPERATIONS: Question 7

Simplify 75 - 27 + 48

What would you like to do now?

Go to full solution

Go to Comments

Go to Algebraic Ops Menu

Reveal answer only

EXIT

Get hint

Page 69: INTERMEDIATE 2 – ADDITIONAL QUESTION BANK UNIT 3 : Further Trig Algebraic Operations Quadratic Functions EXIT.

Find the largest “perfect square” factor of each of

the numbers! Collect identical

surds in same way as you would

letters. .

ALGEBRAIC OPERATIONS: Question 7

Simplify 75 - 27 + 48

What would you like to do now?

Go to full solution

Go to Comments

Go to Algebraic Ops Menu

Reveal answer only

EXIT

Page 70: INTERMEDIATE 2 – ADDITIONAL QUESTION BANK UNIT 3 : Further Trig Algebraic Operations Quadratic Functions EXIT.

ALGEBRAIC OPERATIONS: Question 7

Simplify 75 - 27 + 48 = 63

What would you like to do now?

Try another like this

Go to full solution

Go to Comments

Go to Algebraic Ops MenuEXIT

Page 71: INTERMEDIATE 2 – ADDITIONAL QUESTION BANK UNIT 3 : Further Trig Algebraic Operations Quadratic Functions EXIT.

Question 7 1. Find the largest “perfect square”

factor of each of the numbers!

2. Collect identical surds in same way as you would letters.

Simplify

75 - 27 + 48 = 25 x 3 - 9 x 3 + 16 x 3

= 53 - 33 + 43

= 63 5x – 3x + 4x = 6x

75 - 27 + 48

Comments

Begin Solution

Menu

Back to Home

Try another like this

Page 72: INTERMEDIATE 2 – ADDITIONAL QUESTION BANK UNIT 3 : Further Trig Algebraic Operations Quadratic Functions EXIT.

Comments

Learn Laws of Indices:

1. Find the largest “perfect square” factor of each of the numbers!

2. Collect identical surds in same way as you would letters.

= 25 x 3 - 9 x 3 + 16 x 3

= 53 - 33 + 43

= 63

5x – 3x + 4x = 6x

. .a b a b

e.g. 75 25.3

5 3

25. 3

Menu

Back to Home

Next Comment

Page 73: INTERMEDIATE 2 – ADDITIONAL QUESTION BANK UNIT 3 : Further Trig Algebraic Operations Quadratic Functions EXIT.

Comments

1. Find the largest “perfect square” factor of each of the numbers!

2. Collect identical surds in same way as you would letters.

= 25 x 3 - 9 x 3 + 16 x 3

= 53 - 33 + 43

= 63

5x – 3x + 4x = 6x

The key to these simplificationquestions is that all of theindividual terms can be reduced to a multiple of the same basic surd.

So check that once you have taken “perfect squares” all terms have the same basic surd. If not you may not have used thehighest perfect square for a term.

Menu

Back to Home

Try another

Page 74: INTERMEDIATE 2 – ADDITIONAL QUESTION BANK UNIT 3 : Further Trig Algebraic Operations Quadratic Functions EXIT.

ALGEBRAIC OPERATIONS: Question 7B

Simplify 80 + 45 - 180

What would you like to do now?

Go to full solution

Go to Comments

Go to Algebraic Ops Menu

Reveal answer only

EXIT

Get hint

Page 75: INTERMEDIATE 2 – ADDITIONAL QUESTION BANK UNIT 3 : Further Trig Algebraic Operations Quadratic Functions EXIT.

ALGEBRAIC OPERATIONS: Question 7B

Simplify 80 + 45 - 180

Find the largest “perfect square” factor of each of

the numbers! Collect identical

surds in same way as you would

letters. .

What would you like to do now?

Go to full solution

Go to Comments

Go to Algebraic Ops Menu

Reveal answer only

EXIT

Page 76: INTERMEDIATE 2 – ADDITIONAL QUESTION BANK UNIT 3 : Further Trig Algebraic Operations Quadratic Functions EXIT.

ALGEBRAIC OPERATIONS: Question 7B

Simplify 80 + 45 - 180 = 5

What would you like to do now?

Try another like this

Go to full solution

Go to Comments

Go to Algebraic Ops MenuEXIT

Page 77: INTERMEDIATE 2 – ADDITIONAL QUESTION BANK UNIT 3 : Further Trig Algebraic Operations Quadratic Functions EXIT.

Question 7B

Simplify

80 + 45 - 180

1. Find the largest “perfect square” factor of each of the numbers!

2. Collect identical surds in same way as you would letters.

= 16 x 5 +

80 + 45 - 180

9 x 5 - 36 x 5

= 45 + 35 - 65

= 5 4x + 3x – 6x = x

Comments

Begin Solution

Menu

Back to Home

Try another like thisWhat would you like to do now?

Page 78: INTERMEDIATE 2 – ADDITIONAL QUESTION BANK UNIT 3 : Further Trig Algebraic Operations Quadratic Functions EXIT.

Comments

Learn Laws of Indices:

1. Find the largest “perfect square” factor of each of the numbers!

2. Collect identical surds in same way as you would letters.

= 16 x 5 +

80 + 45 - 180

9 x 5 - 36 x 5

= 45 + 35 - 65

= 5

4x + 3x – 6x = x

. .a b a b

80 16.5

4 5

16. 5

e.g.

Menu

Back to Home

Next Comment

Page 79: INTERMEDIATE 2 – ADDITIONAL QUESTION BANK UNIT 3 : Further Trig Algebraic Operations Quadratic Functions EXIT.

Comments

1. Find the largest “perfect square” factor of each of the numbers!

2. Collect identical surds in same way as you would letters.

= 16 x 5 +

80 + 45 - 180

9 x 5 - 36 x 5

= 45 + 35 - 65

= 5

4x + 3x – 6x = x

The key to these simplificationquestions is that all of theindividual terms can be reduced to a multiple of the same basic surd.

So check that once you have taken “perfect squares” all terms have the same basic surd. If not you may not have used thehighest perfect square for a term.

Menu

Back to Home

Try another

Page 80: INTERMEDIATE 2 – ADDITIONAL QUESTION BANK UNIT 3 : Further Trig Algebraic Operations Quadratic Functions EXIT.

ALGEBRAIC OPERATIONS: Question 7C

Simplify 75 - 27 + 48 300 - 12

What would you like to do now?

Go to full solution

Go to Comments

Go to Algebraic Ops Menu

Reveal answer only

EXIT

Get hint

Page 81: INTERMEDIATE 2 – ADDITIONAL QUESTION BANK UNIT 3 : Further Trig Algebraic Operations Quadratic Functions EXIT.

Deal with top and bottom lines separately. For each, find the

largest “perfect square” factor of

each of the numbers!

ALGEBRAIC OPERATIONS: Question 7C

Simplify 75 - 27 + 48 300 - 12

Now bring both parts together

again. Cancel out where the same basic surd is in

evidence on both top and bottom.

What would you like to do now?

Go to full solution

Go to Comments

Go to Algebraic Ops Menu

Reveal answer only

EXIT

Page 82: INTERMEDIATE 2 – ADDITIONAL QUESTION BANK UNIT 3 : Further Trig Algebraic Operations Quadratic Functions EXIT.

ALGEBRAIC OPERATIONS: Question 7C

Simplify 75 - 27 + 48 300 - 12

3/4=

What would you like to do now?

Go to full solution

Go to Comments

Go to Algebraic Ops MenuEXIT

Page 83: INTERMEDIATE 2 – ADDITIONAL QUESTION BANK UNIT 3 : Further Trig Algebraic Operations Quadratic Functions EXIT.

Question 7C 1. Deal with top and bottom lines separately. For each, find the largest “perfect square” factor of each of the numbers!

5x - 3x + 4x = 6x

Simplify

75 - 27 + 48

300 - 12 = 25 x 3 -

75 - 27 + 48

= 53 - 33 + 43

= 63

Top Line:

9 x 3 + 16 x 3

Comments

Begin Solution

Continue Solution

Menu

Back to Home

Page 84: INTERMEDIATE 2 – ADDITIONAL QUESTION BANK UNIT 3 : Further Trig Algebraic Operations Quadratic Functions EXIT.

Question 7C 1. Deal with top and bottom lines seperately. For each, find the largest “perfect square” factor of each of the numbers!

10x - 2x = 8x

Simplify

75 - 27 + 48

300 - 12

Bottom Line:

300 - 12

= 100 x 3 -

= 103 - 23

= 83

4 x 3

Comments

Begin Solution

Continue Solution

Menu

Back to Home

Page 85: INTERMEDIATE 2 – ADDITIONAL QUESTION BANK UNIT 3 : Further Trig Algebraic Operations Quadratic Functions EXIT.

Question 7C 2. Now bring both parts together again. Cancel out where the same basic surd is in evidence on both top and bottom.

Simplify

75 - 27 + 48

300 - 12

75 - 27 + 48

300 - 12

6383

3/4

=

=

3

4

Comments

Begin Solution

Continue Solution

Menu

Back to Home

What would you like to do now?

Page 86: INTERMEDIATE 2 – ADDITIONAL QUESTION BANK UNIT 3 : Further Trig Algebraic Operations Quadratic Functions EXIT.

Comments

Learn Laws of Indices:

. .a b a b

80 16.5

4 5

16. 5

e.g.

1. Deal with top and bottom lines seperately. For each, find the largest “perfect square” factor of each of the numbers!

5x - 3x + 4x = 6x

= 25 x 3 -

75 - 27 + 48

= 53 - 33 + 43

= 63

Top Line:

9 x 3 + 16 x 3

Menu

Back to Home

Next Comment

Page 87: INTERMEDIATE 2 – ADDITIONAL QUESTION BANK UNIT 3 : Further Trig Algebraic Operations Quadratic Functions EXIT.

Comments

In questions involving fractions always treat each line separately.Reduce each line to its simplest form before dividing.

2. Now bring both parts together again. Cancel out where the same basic surd is in evidence on both top and bottom.

75 - 27 + 48

300 - 12

6383

3/4

=

=

The key to these simplificationquestions is that all of theindividual terms can be reduced to a multiple of the same basic surd. In fractions these will thenoften cancel.

Remember to cancel numbers too!!

Menu

Back to Home

Next Comment

Page 88: INTERMEDIATE 2 – ADDITIONAL QUESTION BANK UNIT 3 : Further Trig Algebraic Operations Quadratic Functions EXIT.

ALGEBRAIC OPERATIONS: Question 8

12

6Express with a rational denominator.

What would you like to do now?

Go to full solution

Go to Comments

Go to Algebraic Ops Menu

Reveal answer only

EXIT

Get hint

Page 89: INTERMEDIATE 2 – ADDITIONAL QUESTION BANK UNIT 3 : Further Trig Algebraic Operations Quadratic Functions EXIT.

To change the look of a fraction but

not the value multiply top & bottom by the

same amount ie 6.

ALGEBRAIC OPERATIONS: Question 8

12

6Express with a rational denominator.

a a a

What would you like to do now?

Go to full solution

Go to Comments

Go to Algebraic Ops Menu

Reveal answer only

EXIT

Page 90: INTERMEDIATE 2 – ADDITIONAL QUESTION BANK UNIT 3 : Further Trig Algebraic Operations Quadratic Functions EXIT.

ALGEBRAIC OPERATIONS: Question 8

= 26

12

6Express with a rational denominator.

What would you like to do now?

Try another like this

Go to full solution

Go to Comments

Go to Algebraic Ops MenuEXIT

Page 91: INTERMEDIATE 2 – ADDITIONAL QUESTION BANK UNIT 3 : Further Trig Algebraic Operations Quadratic Functions EXIT.

Question 8 1. To change the look of a fraction but not the value multiply top & bottom by the same amount ie 6.12

6Express

with a rational

denominator.

12

6 =12

6x 6

6

126

6=

= 26

a a a

2

1

Comments

Begin Solution

Menu

Back to Home

Try another like this

Page 92: INTERMEDIATE 2 – ADDITIONAL QUESTION BANK UNIT 3 : Further Trig Algebraic Operations Quadratic Functions EXIT.

Comments

Learn Laws of Surds:

Rationalising the denominator:

1

a Required to remove from the denominator.

Multiply top and bottom by a

1.a a

aa a

1. To change the look of a fraction but not the value multiply top & bottom by the same amount ie 6.

12

6 =12

6x 6

6

126

6=

= 26

a a a

Menu

Back to Home

Next Comment

Page 93: INTERMEDIATE 2 – ADDITIONAL QUESTION BANK UNIT 3 : Further Trig Algebraic Operations Quadratic Functions EXIT.

Comments

Learn Laws of Surds:

e.g.

Rationalising the denominator:1. To change the look of a fraction but not the value multiply top & bottom by the same amount ie 6.

12

6 =12

6x 6

6

126

6=

= 26

a a a

1 1 5 5.

55 5 5

Menu

Back to Home

Try another

Page 94: INTERMEDIATE 2 – ADDITIONAL QUESTION BANK UNIT 3 : Further Trig Algebraic Operations Quadratic Functions EXIT.

ALGEBRAIC OPERATIONS: Question 8B

843

Simplify

What would you like to do now?

Go to full solution

Go to Comments

Go to Algebraic Ops Menu

Reveal answer only

EXIT

Get hint

Page 95: INTERMEDIATE 2 – ADDITIONAL QUESTION BANK UNIT 3 : Further Trig Algebraic Operations Quadratic Functions EXIT.

ALGEBRAIC OPERATIONS: Question 8B

Find the largest “perfect square” factor of each of

the numbers! Collect identical

surds in same way as you would

letters. .

843

Simplify

What would you like to do now?

Go to full solution

Go to Comments

Go to Algebraic Ops Menu

Reveal answer only

EXIT

Page 96: INTERMEDIATE 2 – ADDITIONAL QUESTION BANK UNIT 3 : Further Trig Algebraic Operations Quadratic Functions EXIT.

ALGEBRAIC OPERATIONS: Question 8B

= 27 843

Simplify

What would you like to do now?

Try another like this

Go to full solution

Go to Comments

Go to Algebraic Ops MenuEXIT

Page 97: INTERMEDIATE 2 – ADDITIONAL QUESTION BANK UNIT 3 : Further Trig Algebraic Operations Quadratic Functions EXIT.

Question 8B 1. Re-write expression using laws of surds

843

Simplify

= 27

843 =

84 3

= 28

= 4 x 7

a a

bb

. .a b a b

Comments

Begin Solution

Menu

Back to Home

Try another like this

What would you like to do now?

Page 98: INTERMEDIATE 2 – ADDITIONAL QUESTION BANK UNIT 3 : Further Trig Algebraic Operations Quadratic Functions EXIT.

Comments

Learn Laws of Surds:

e.g.

1. Re-write expression using laws of surds

= 27

843 =

84 3

= 28

= 4 x 7

. .a b a b

a a

bb

a a

bb

24 244 2

66

Menu

Back to Home

Next Comment

Page 99: INTERMEDIATE 2 – ADDITIONAL QUESTION BANK UNIT 3 : Further Trig Algebraic Operations Quadratic Functions EXIT.

Comments

Learn Laws of Surds:

e.g.

1. Re-write expression using laws of surds

= 27

843 =

84 3

= 28

= 4 x 7

. .a b a b

a a

bb

. .a b a b

75 25.3 25. 3 5 3

Menu

Back to Home

Try another

Page 100: INTERMEDIATE 2 – ADDITIONAL QUESTION BANK UNIT 3 : Further Trig Algebraic Operations Quadratic Functions EXIT.

ALGEBRAIC OPERATIONS: Question 8C

Find the value of tanx° giving your answer as a fraction with a rational denominator.

3m

62mWhat would you like to do now?

Go to full solution

Go to Comments

Go to Algebraic Ops Menu

Reveal answer only

EXIT

Get hint

Page 101: INTERMEDIATE 2 – ADDITIONAL QUESTION BANK UNIT 3 : Further Trig Algebraic Operations Quadratic Functions EXIT.

To change the look of a fraction but not the value multiply top & bottom by the same

amount ie 2.

ALGEBRAIC OPERATIONS: Question 8C

When dealing with fractions check that you have

simplified as far as possible.

Find the value of tanx° giving your answer as a fraction with a rational denominator.

3m

62m

0tanopp

xadj

What would you like to do now?

Go to full solution

Go to Comments

Go to Algebraic Ops Menu

Reveal answer only

EXIT

Page 102: INTERMEDIATE 2 – ADDITIONAL QUESTION BANK UNIT 3 : Further Trig Algebraic Operations Quadratic Functions EXIT.

ALGEBRAIC OPERATIONS: Question 8C

Find the value of tanx° giving your answer as a fraction with a rational denominator.

3m

62m

2

4=

What would you like to do now?

Go to full solution

Go to Comments

Go to Algebraic Ops MenuEXIT

Page 103: INTERMEDIATE 2 – ADDITIONAL QUESTION BANK UNIT 3 : Further Trig Algebraic Operations Quadratic Functions EXIT.

Question 8C

Find the value of tanx°

giving your answer as

a fraction with a

rational denominator.

3m

62m

1. To change the look of a fraction but not the value multiply top & bottom by the same amount ie 2.

tanx° = 3

62

=3

62x 2

2

32

6x2=

32

12=

2

4=

0tanopp

xadj

Simplify!!!1

4

Comments

Begin Solution

Continue Solution

Menu

Back to HomeWhat would you like to do now?

Page 104: INTERMEDIATE 2 – ADDITIONAL QUESTION BANK UNIT 3 : Further Trig Algebraic Operations Quadratic Functions EXIT.

Comments

Learn Laws of Surds:1. To change the look of a fraction

but not the value multiply top & bottom by the same amount ie 2.

tanx° = 3

62

=3

62x 2

2

32

6x2=

32

12=

2

4=

Rationalising the denominator:

1

a Required to remove from the denominator.

Multiply top and bottom by a

1.a a

aa a

Menu

Back to Home

Next Comment

Page 105: INTERMEDIATE 2 – ADDITIONAL QUESTION BANK UNIT 3 : Further Trig Algebraic Operations Quadratic Functions EXIT.

Comments

Learn Laws of Surds:

e.g.

1. To change the look of a fraction but not the value multiply top & bottom by the same amount ie 2.

tanx° = 3

62

=3

62x 2

2

32

6x2=

32

12=

2

4=

2 2 3 2 3.

33 3 3

In questions involving fractionsmake sure you have simplified as far as possible.

Simplify!!!

Menu

Back to Home

Next Comment

Page 106: INTERMEDIATE 2 – ADDITIONAL QUESTION BANK UNIT 3 : Further Trig Algebraic Operations Quadratic Functions EXIT.

INTERMEDIATE 2 – ADDITIONAL QUESTION BANK

UNIT 3 :Further Trig

You have chosen to study:

Please choose a question to attempt from the following:

1 2

EXITBack to

Unit 3 Menu

Page 107: INTERMEDIATE 2 – ADDITIONAL QUESTION BANK UNIT 3 : Further Trig Algebraic Operations Quadratic Functions EXIT.

Further Trig : Question 1

The graph below shows a curve with equation in the form y = asinbx° .

Write down the values of a and b.

1200 2400 3600

y = asinbx°

EXIT

Go to full solution

Go to Comments

Further Trig Menu

Reveal answer only

Get hint

What would you like to do now?

Page 108: INTERMEDIATE 2 – ADDITIONAL QUESTION BANK UNIT 3 : Further Trig Algebraic Operations Quadratic Functions EXIT.

Further Trig : Question 1

The graph below shows a curve with equation in the form y = asinbx° .

Write down the values of a and b.

1200 2400 3600

y = asinbx°

a = amplitude = ½ vertical extent. b = no. of waves in 3600

EXIT

Go to full solution

Go to Comments

Further Trig Menu

Reveal answer only

What would you like to do now?

Page 109: INTERMEDIATE 2 – ADDITIONAL QUESTION BANK UNIT 3 : Further Trig Algebraic Operations Quadratic Functions EXIT.

Further Trig : Question 1

The graph below shows a curve with equation in the form y = asinbx° .

Write down the values of a and b.

1200 2400 3600

y = asinbx°

a = 2 b = 3

EXIT

Go to full solution

Go to Comments

Further Trig Menu

What would you like to do now?

Try another like this

Page 110: INTERMEDIATE 2 – ADDITIONAL QUESTION BANK UNIT 3 : Further Trig Algebraic Operations Quadratic Functions EXIT.

Question 1

1200 2400 3600

y = asinbx°

Write down the values of a and b.

1. a = amplitude = ½ vertical extent.

max /min = ±2 so a = 2

2. b = no. of waves in 3600

3 complete waves from 0 to 360

so b = 3

Comments

Begin Solution

Menu

Back to Home

Try another like this

What would you like to do now?

Page 111: INTERMEDIATE 2 – ADDITIONAL QUESTION BANK UNIT 3 : Further Trig Algebraic Operations Quadratic Functions EXIT.

Comments

1. a = amplitude = ½ vertical extent.

max /min = ±2 so a = 2

2. b = no. of waves in 3600

3 complete waves from 0 to 360

so b = 3

Learn basic trig graphs:

i.e. y = sinx˚

Max. = 1Min. = -1One cycle in 360˚

360

-1

1

x

y

Menu

Back to Home

Next Comment

Page 112: INTERMEDIATE 2 – ADDITIONAL QUESTION BANK UNIT 3 : Further Trig Algebraic Operations Quadratic Functions EXIT.

Comments

1. a = amplitude = ½ vertical extent.

max /min = ±2 so a = 2

2. b = no. of waves in 3600

3 complete waves from 0 to 360

so b = 3

Learn basic trig graphs:

i.e. y = sinx˚

360

-1

1

x

y

y = asinx˚

Stretch factor

y = sinbx˚

Number of cycles in 360˚

One cycle

Menu

Back to Home

Try another

Page 113: INTERMEDIATE 2 – ADDITIONAL QUESTION BANK UNIT 3 : Further Trig Algebraic Operations Quadratic Functions EXIT.

Further Trig : Question 1B

The graph below shows a curve with equation in the form y = acosbx° .

Write down the values of a and b.

y = acosbx°

900 1800 36002700

EXIT

Go to full solution

Go to Comments

Further Trig Menu

Reveal answer only

Get hint

What would you like to do now?

Page 114: INTERMEDIATE 2 – ADDITIONAL QUESTION BANK UNIT 3 : Further Trig Algebraic Operations Quadratic Functions EXIT.

Further Trig : Question 1B

The graph below shows a curve with equation in the form y = acosbx° .

Write down the values of a and b.

y = acosbx°

900 1800 36002700

a = amplitude = ½ vertical extent. b = no. of waves in 3600

EXIT

Go to full solution

Go to Comments

Further Trig Menu

Reveal answer only

What would you like to do now?

Page 115: INTERMEDIATE 2 – ADDITIONAL QUESTION BANK UNIT 3 : Further Trig Algebraic Operations Quadratic Functions EXIT.

Further Trig : Question 1B

The graph below shows a curve with equation in the form y = acosbx° .

Write down the values of a and b.

y = acosbx°

900 1800 36002700

a = 5 b = 2

EXIT

Go to full solution

Go to Comments

Further Trig Menu

What would you like to do now?

Page 116: INTERMEDIATE 2 – ADDITIONAL QUESTION BANK UNIT 3 : Further Trig Algebraic Operations Quadratic Functions EXIT.

y = acosbx°

900 1800 36002700

Question 1B

Write down the values of a and b.

1. a = amplitude = ½ vertical extent.

max /min = ±5 so a = 5

2. b = no. of waves in 3600

2 complete waves from 0 to 360

so b = 2

Comments

Begin Solution

Continue Solution

Menu

Back to Home

What would you like to do now?

Page 117: INTERMEDIATE 2 – ADDITIONAL QUESTION BANK UNIT 3 : Further Trig Algebraic Operations Quadratic Functions EXIT.

Comments

1. a = amplitude = ½ vertical extent.

max /min = ±5 so a = 5

2. b = no. of waves in 3600

2 complete waves from 0 to 360

so b = 2

Learn basic trig graphs:

i.e. y = cosx˚

Max. = 1Min. = -1One cycle in 360˚

360

-1

1

x

y

Menu

Back to Home

Next Comment

Page 118: INTERMEDIATE 2 – ADDITIONAL QUESTION BANK UNIT 3 : Further Trig Algebraic Operations Quadratic Functions EXIT.

360

-1

1

x

y

Comments

1. a = amplitude = ½ vertical extent.

max /min = ±2 so a = 2

2. b = no. of waves in 3600

3 complete waves from 0 to 360

so b = 3

Learn basic trig graphs:

i.e. y = cosx˚

y = acosx˚

Stretch factor

y = cosbx˚

Number of cycles in 360˚

One cycle

Menu

Back to Home

Page 119: INTERMEDIATE 2 – ADDITIONAL QUESTION BANK UNIT 3 : Further Trig Algebraic Operations Quadratic Functions EXIT.

Further Trig : Question 2

Solve 5sinx° - 1 = 1 where 0<x<360

EXIT

Go to full solution

Go to Comments

Further Trig Menu

Reveal answer only

Get hint

What would you like to do now?

Page 120: INTERMEDIATE 2 – ADDITIONAL QUESTION BANK UNIT 3 : Further Trig Algebraic Operations Quadratic Functions EXIT.

Further Trig : Question 2

Solve 5sinx° - 1 = 1 where 0<x<360

Always re-arrange given equation to:

sin x =cosx =tan x=

Use CAST diagram to decide which quadrants

angles lie.

Remember that answers to trig equations come

in pairs.

EXIT

Go to full solution

Go to Comments

Further Trig Menu

Reveal answer only

What would you like to do now?

Page 121: INTERMEDIATE 2 – ADDITIONAL QUESTION BANK UNIT 3 : Further Trig Algebraic Operations Quadratic Functions EXIT.

Further Trig : Question 2

Solve 5sinx° - 1 = 1 where 0<x<360

x = 23.6°

x = 156.4°

EXIT

Go to full solution

Go to Comments

Further Trig Menu

What would you like to do now?

Try another like this

Page 122: INTERMEDIATE 2 – ADDITIONAL QUESTION BANK UNIT 3 : Further Trig Algebraic Operations Quadratic Functions EXIT.

Question 2 1. Always re-arrange to sinx0 = …...

2. Use the CAST diagram to decide which quadrant angles lie in.

Solve

5sinx° - 1 = 1

where 0<x<360

sinx = 0.4

5sinx° - 1 = 1

5sinx° = 2

180 -

180 + 360 -

sin all

tan cos

Q1 or Q2

Where is sin

positive?

Comments

Begin Solution

Continue Solution

Menu

Back to Home

Page 123: INTERMEDIATE 2 – ADDITIONAL QUESTION BANK UNIT 3 : Further Trig Algebraic Operations Quadratic Functions EXIT.

Question 2 1. Always re-arrange to sinx0 = …...

3. Calculate angles remembering that answers to trig equations come in pairs.

Solve

5sinx° - 1 = 1

where 0<x<360

sinx = 0.4

5sinx° - 1 = 1

5sinx° = 2

Q1 or Q2

sin-1 0.4 = 23.6

Q1: angle = 23.6°

Q2: angle = 180° - 23.6°

= 156.4°Comments

Begin Solution

Menu

Back to Home

Try another like this

What would you like to do now?

Page 124: INTERMEDIATE 2 – ADDITIONAL QUESTION BANK UNIT 3 : Further Trig Algebraic Operations Quadratic Functions EXIT.

Comments

We are finding the two angles at which the sine curve reaches a height of 0.4

90 180 270 360

-1

-0.5

0.5

1

x

y

0.4

23.6 156.4

1. Always re-arrange to sinx0 = …...

3. Calculate angles remembering that answers to trig equations come in pairs.

sinx = 0.4

5sinx° - 1 = 1

5sinx° = 2

Q1 or Q2

Q2: angle = 180° - 23.6°

sin-1 0.4 = 23.6

Q1: angle = 23.6°

= 156.4°Menu

Back to Home

Try another

Page 125: INTERMEDIATE 2 – ADDITIONAL QUESTION BANK UNIT 3 : Further Trig Algebraic Operations Quadratic Functions EXIT.

Further Trig : Question 2B

Solve 3tanx° + 7 = 2 where 0<x<360

EXIT

Go to full solution

Go to Comments

Further Trig Menu

Reveal answer only

Get hint

What would you like to do now?

Page 126: INTERMEDIATE 2 – ADDITIONAL QUESTION BANK UNIT 3 : Further Trig Algebraic Operations Quadratic Functions EXIT.

Further Trig : Question 2B

Solve 3tanx° + 7 = 2 where 0<x<360

Always re-arrange given equation to:

sin x =cosx =tan x=

Use CAST diagram to decide which quadrants

angles lie.

Remember that answers to trig equations come

in pairs.

EXIT

Go to full solution

Go to Comments

Further Trig Menu

Reveal answer only

What would you like to do now?

Page 127: INTERMEDIATE 2 – ADDITIONAL QUESTION BANK UNIT 3 : Further Trig Algebraic Operations Quadratic Functions EXIT.

Solve 3tanx° + 7 = 2 where 0<x<360

Further Trig : Question 2B

x = 121.0°

x = 301.0°

EXIT

Go to full solution

Go to Comments

Further Trig Menu

What would you like to do now?

Page 128: INTERMEDIATE 2 – ADDITIONAL QUESTION BANK UNIT 3 : Further Trig Algebraic Operations Quadratic Functions EXIT.

Question 2B

Solve

3tanx° + 7 = 2

where 0<x<360

1. Always re-arrange to tanx0 = …...

tanx = -5/3

3tanx° + 7 = 2

3tanx° = -5

2. Use the CAST diagram to decide which quadrant angles lie in.

180 -

180 + 360 -

sin all

tan cos

Q2 or Q4

Where is tan

negative?

Comments

Begin Solution

Continue Solution

Menu

Back to Home

Page 129: INTERMEDIATE 2 – ADDITIONAL QUESTION BANK UNIT 3 : Further Trig Algebraic Operations Quadratic Functions EXIT.

Question 2B

Solve

3tanx° + 7 = 2

where 0<x<360

1. Always re-arrange to tanx0 = …...

tanx = -5/3

3tanx° + 7 = 2

3tanx° = -5

Q2 or Q4

tan-1 (53) = 59.0

Q2: angle = 180° - 59.0°

Q4: angle = 360° - 59.0°

3. Calculate angles remembering that answers to trig equations come in pairs.

= 301.0°

= 121.0°

Comments

Begin Solution

Continue Solution

Menu

Back to Home

What would you like to do now?

Page 130: INTERMEDIATE 2 – ADDITIONAL QUESTION BANK UNIT 3 : Further Trig Algebraic Operations Quadratic Functions EXIT.

Comments

1. Always re-arrange to tanx0 = …...

tanx = -5/3

3tanx° + 7 = 2

3tanx° = -5

tan-1 (53) = 59.0

Q2: angle = 180° - 59.0°

Q4: angle = 360° - 59.0°

3. Calculate angles remembering that answers to trig equations come in pairs.

= 301.0°

= 121.0°

We are finding the two angles at which the tan graph reaches a height of 5

3

90 180 270 360

-3

-2

-1

1

2

3

x

y

121.0 301.0

5

3

Menu

Back to Home

Next Comment

Page 131: INTERMEDIATE 2 – ADDITIONAL QUESTION BANK UNIT 3 : Further Trig Algebraic Operations Quadratic Functions EXIT.

Comments

1. Always re-arrange to tanx0 = …...

tanx = -5/3

3tanx° + 7 = 2

3tanx° = -5

tan-1 (53) = 59.0

Q2: angle = 180° - 59.0°

Q4: angle = 360° - 59.0°

3. Calculate angles remembering that answers to trig equations come in pairs.

= 301.0°

= 121.0°

1 5tan

3x

Note:

Never put a negative value into the

inverse trig function

DO:

then CAST diagram

Menu

Back to Home

Next Comment

Page 132: INTERMEDIATE 2 – ADDITIONAL QUESTION BANK UNIT 3 : Further Trig Algebraic Operations Quadratic Functions EXIT.

INTERMEDIATE 2 – ADDITIONAL QUESTION BANK

UNIT 3 : Quadratic Functions

You have chosen to study:

Please choose a question to attempt from the following:

1 2

EXITBack to

Unit 3 Menu

3 4

Page 133: INTERMEDIATE 2 – ADDITIONAL QUESTION BANK UNIT 3 : Further Trig Algebraic Operations Quadratic Functions EXIT.

Quadratic Functions: Question 1

Solve the equation x2 – 6x + 7 = 0 giving your answers to 2 decimal places.

EXIT

Go to full solution

Go to Comments

Go to Quadratics Menu

Reveal answer only

Get hint

What would you like to do now?

Page 134: INTERMEDIATE 2 – ADDITIONAL QUESTION BANK UNIT 3 : Further Trig Algebraic Operations Quadratic Functions EXIT.

Quadratic Functions: Question 1

Solve the equation x2 – 6x + 7 = 0 giving your answers to 2 decimal places.

Write down values of a,

b & c. Evaluate b2 – 4ac .

Now use quadratic formula,

remembering that you should get

two answers.

Remember to round.

EXIT

Go to full solution

Go to Comments

Go to Quadratics Menu

Reveal answer only

What would you like to do now?

Page 135: INTERMEDIATE 2 – ADDITIONAL QUESTION BANK UNIT 3 : Further Trig Algebraic Operations Quadratic Functions EXIT.

Quadratic Functions: Question 1

Solve the equation x2 – 6x + 7 = 0 giving your answers to 2 decimal places.

x = 4.41 or 1.59

EXIT

Go to full solution

Go to Comments

Go to Quadratics Menu

Try another like this

What would you like to do now?

Page 136: INTERMEDIATE 2 – ADDITIONAL QUESTION BANK UNIT 3 : Further Trig Algebraic Operations Quadratic Functions EXIT.

Question 1

Solve the equation

x2 – 6x + 7 = 0

giving your answers

to 2 decimal places.

1. Write down values of a, b & c.

x2 – 6x + 7 = 0

a = 1 b = (-6) c = 7

2. Evaluate b2 – 4ac .

b2 – 4ac = (-6)2 – (4x1x7)

= 36 – 28

= 8

Comments

Begin Solution

Continue Solution

Menu

Back to Home

Page 137: INTERMEDIATE 2 – ADDITIONAL QUESTION BANK UNIT 3 : Further Trig Algebraic Operations Quadratic Functions EXIT.

Question 1

Solve the equation

x2 – 6x + 7 = 0

giving your answers

to 2 decimal places.

1. Write down values of a, b & c.

x2 – 6x + 7 = 0

a = 1 b = -6 c = 7

3. Now use quadratic formula.

x = -b ± (b2 – 4ac )

2a

x = 6 ± 8

24. Rewrite with brackets and now use

calculator.

= (6 + 8) 2 or (6 - 8) 2

Comments

Begin Solution

Continue Solution

Menu

Back to Home

Page 138: INTERMEDIATE 2 – ADDITIONAL QUESTION BANK UNIT 3 : Further Trig Algebraic Operations Quadratic Functions EXIT.

Question 1

Solve the equation

x2 – 6x + 7 = 0

giving your answers

to 2 decimal places.

1. Write down values of a, b & c.

x2 – 6x + 7 = 0

a = 1 b = -6 c = 7

4. Rewrite with brackets and now use calculator.

= (6 + 8) 2 or (6 - 8) 2 = 4.414… or 1.585..

5. Remember to round.

x = 4.41 or 1.59Comments

Begin Solution

Try another like this

Menu

Back to Home

What would you like to do now?

Page 139: INTERMEDIATE 2 – ADDITIONAL QUESTION BANK UNIT 3 : Further Trig Algebraic Operations Quadratic Functions EXIT.

Comments

1. Write down values of a, b & c.

x2 – 6x + 7 = 0

a = 1 b = -6 c = 7

3. Now use quadratic formula.

x = -b ± (b2 – 4ac )

2a

x = 6 ± 8

24. Rewrite with brackets and now use

calculator.

= (6 + 8) 2 or (6 - 8) 2

For any question involving a quadratic in which you are asked to give your answer to a given number of decimal places

OR

a given number of significant figures:

THINK QUADRATIC FORMULA!!

Menu

Back to Home

Next Comment

Page 140: INTERMEDIATE 2 – ADDITIONAL QUESTION BANK UNIT 3 : Further Trig Algebraic Operations Quadratic Functions EXIT.

Comments

1. Write down values of a, b & c.

x2 – 6x + 7 = 0

a = 1 b = -6 c = 7

3. Now use quadratic formula.

x = -b ± (b2 – 4ac )

2a

x = 6 ± 8

24. Rewrite with brackets and now use

calculator.

= (6 + 8) 2 or (6 - 8) 2

ax2 + bx + c = 0

Refer to the Formula Sheet:

x = -b ± (b2 – 4ac )

2a

Quadratic Formula:

Menu

Back to Home

Next Comment

Page 141: INTERMEDIATE 2 – ADDITIONAL QUESTION BANK UNIT 3 : Further Trig Algebraic Operations Quadratic Functions EXIT.

Comments

1. Write down values of a, b & c.

x2 – 6x + 7 = 0

a = 1 b = -6 c = 7

3. Now use quadratic formula.

x = -b ± (b2 – 4ac )

2a

x = 6 ± 8

24. Rewrite with brackets and now use

calculator.

= (6 + 8) 2 or (6 - 8) 2

Take care when allocating a

value to a, b and c.

ax2 + bx + c = 0

1x2 - 6x + 7 = 0

a =1b = (-6)c = 7

Menu

Back to Home

Next Comment

Page 142: INTERMEDIATE 2 – ADDITIONAL QUESTION BANK UNIT 3 : Further Trig Algebraic Operations Quadratic Functions EXIT.

Note:

Finding first eases working.

Bracket all negative numbers. Watch out for double negative!

Comments

1. Write down values of a, b & c.

x2 – 6x + 7 = 0

a = 1 b = -6 c = 7

b2 – 4ac

2. Evaluate b2 – 4ac .

b2 – 4ac = (-6)2 – (4x1x7)

= 36 – 28

= 8

Menu

Back to Home

Try another

Page 143: INTERMEDIATE 2 – ADDITIONAL QUESTION BANK UNIT 3 : Further Trig Algebraic Operations Quadratic Functions EXIT.

Quadratic Functions: Question 1B

Go to full solution

Go to Comments

Go to Quadratics Menu

Reveal answer only

EXIT

Get hint

Solve the equation 3x2 + 5x - 1 = 0 giving your answers to 2 significant figures.

What would you like to do now?

Page 144: INTERMEDIATE 2 – ADDITIONAL QUESTION BANK UNIT 3 : Further Trig Algebraic Operations Quadratic Functions EXIT.

Quadratic Functions: Question 1B

Go to full solution

Go to Comments

Go to Quadratics Menu

Reveal answer only

EXIT

Write down values of a,

b & c. Evaluate b2 – 4ac .

Now use quadratic formula,

remembering that you should get

two answers.

Remember to round.

Solve the equation 3x2 + 5x - 1 = 0 giving your answers to 2 significant figures.

What would you like to do now?

Page 145: INTERMEDIATE 2 – ADDITIONAL QUESTION BANK UNIT 3 : Further Trig Algebraic Operations Quadratic Functions EXIT.

Quadratic Functions: Question 1B

Go to full solution

Go to Comments

Go to Quadratics Menu

EXIT

x = 0.18 or -1.8

Solve the equation 3x2 + 5x - 1 = 0 giving your answers to 2 significant figures.

What would you like to do now?

Page 146: INTERMEDIATE 2 – ADDITIONAL QUESTION BANK UNIT 3 : Further Trig Algebraic Operations Quadratic Functions EXIT.

Comments

Begin Solution

Continue Solution

Question 1B

Quadratics Menu

Back to Home

1. Write down values of a, b & c.

3x2 + 5x – 1 = 0

a = 3 b = 5 c = (-1)

2. Evaluate b2 – 4ac .

b2 – 4ac = (5)2 – (4 x 3 x -1)

= 25 – (- 12)

= 37

Solve the equation

3x2 + 5x - 1 = 0

giving your answers

to 2 significant figures.

Page 147: INTERMEDIATE 2 – ADDITIONAL QUESTION BANK UNIT 3 : Further Trig Algebraic Operations Quadratic Functions EXIT.

Comments

Begin Solution

Continue Solution

Question 1B

Quadratics Menu

Back to Home

1. Write down values of a, b & c.

3. Now use quadratic formula.

x = -b ± (b2 – 4ac )

2a

x = -5 ± 37

24. Rewrite with brackets and now use

calculator.

= (-5 + 37) 2 or (-5 - 37) 2

3x2 + 5x – 1 = 0

a = 3 b = 5 c = (-1)

Solve the equation

3x2 + 5x - 1 = 0

giving your answers

to 2 significant figures.

Page 148: INTERMEDIATE 2 – ADDITIONAL QUESTION BANK UNIT 3 : Further Trig Algebraic Operations Quadratic Functions EXIT.

Comments

Begin Solution

Continue Solution

Question 1B

Quadratics Menu

Back to Home

1. Write down values of a, b & c.

4. Rewrite with brackets and now use calculator.

= (-5 + 37) 2 or (-5 - 37) 2

5. Remember to round.

x = 0.18 or -1.8

3x2 + 5x – 1 = 0

a = 3 b = 5 c = (-1)

Solve the equation

3x2 + 5x - 1 = 0

giving your answers

to 2 significant figures.

= 0.180… or -1.847..

What would you like to do now?

Page 149: INTERMEDIATE 2 – ADDITIONAL QUESTION BANK UNIT 3 : Further Trig Algebraic Operations Quadratic Functions EXIT.

Comments

Quadratics Menu

Back to Home

Next Comment

For any question involving a quadratic in which you are asked to give your answer to a given number of decimal places

OR

a given number of significant figures:

THINK QUADRATIC FORMULA!!

1. Write down values of a, b & c.

3. Now use quadratic formula.

x = -b ± (b2 – 4ac )

2a

x = -5 ± 37

24. Rewrite with brackets and now

use calculator.

= (-5 + 37) 2 or (-5 - 37) 2

3x2 + 5x – 1 = 0

a = 3 b = 5 c = (-1)

Page 150: INTERMEDIATE 2 – ADDITIONAL QUESTION BANK UNIT 3 : Further Trig Algebraic Operations Quadratic Functions EXIT.

Comments

Quadratics Menu

Back to Home

Next Comment

ax2 + bx + c = 0

Refer to the Formula Sheet:

x = -b ± (b2 – 4ac )

2a

Quadratic Formula:

1. Write down values of a, b & c.

3. Now use quadratic formula.

x = -b ± (b2 – 4ac )

2a

x = -5 ± 37

24. Rewrite with brackets and now use

calculator.

= (-5 + 37) 2 or (-5 - 37) 2

3x2 + 5x – 1 = 0

a = 3 b = 5 c = (-1)

Page 151: INTERMEDIATE 2 – ADDITIONAL QUESTION BANK UNIT 3 : Further Trig Algebraic Operations Quadratic Functions EXIT.

Comments

Quadratics Menu

Back to Home

Next Comment

Take care when allocating a

value to a, b and c.

ax2 + bx + c = 0

3x2 + 5x - 1 = 0

a =3b = 5c = -1

1. Write down values of a, b & c.

3. Now use quadratic formula.

x = -b ± (b2 – 4ac )

2a

x = -5 ± 37

24. Rewrite with brackets and now use

calculator.

= (-5 + 37) 2 or (-5 - 37) 2

3x2 + 5x – 1 = 0

a = 3 b = 5 c = (-1)

Page 152: INTERMEDIATE 2 – ADDITIONAL QUESTION BANK UNIT 3 : Further Trig Algebraic Operations Quadratic Functions EXIT.

Note:

Finding first eases working.

Bracket all negative numbers. Watch out for double negative!

Comments

Quadratics Menu

Back to Home

Next Comment

b2 – 4ac1. Write down values of a, b & c.

3x2 + 5x – 1 = 0

a = 3 b = 5 c = (-1)

2. Evaluate b2 – 4ac .

b2 – 4ac = (5)2 – (4 x 3 x -1)

= 25 – (- 12)

= 37

Page 153: INTERMEDIATE 2 – ADDITIONAL QUESTION BANK UNIT 3 : Further Trig Algebraic Operations Quadratic Functions EXIT.

Quadratic Functions: Question 2

Go to full solution

Go to Comments

Go to Quadratics Menu

Reveal answer only

EXIT

Get hint

Solve the equation 3x2 = x + 1 giving your answers to 2 decimal places.

What would you like to do now?

Page 154: INTERMEDIATE 2 – ADDITIONAL QUESTION BANK UNIT 3 : Further Trig Algebraic Operations Quadratic Functions EXIT.

Quadratic Functions: Question 2

Go to full solution

Go to Comments

Go to Quadratics Menu

Reveal answer only

EXIT

Write down values of a,

b & c. Evaluate b2 – 4ac .

Now use quadratic formula,

remembering that you should get

two answers.

Remember to round.

Solve the equation 3x2 = x + 1 giving your answers to 2 decimal places.

Re-write quadratic:

make it equal to

zero before solving.

What would you like to do now?

Page 155: INTERMEDIATE 2 – ADDITIONAL QUESTION BANK UNIT 3 : Further Trig Algebraic Operations Quadratic Functions EXIT.

Quadratic Functions: Question 2

Go to full solution

Go to Comments

Go to Quadratics Menu

EXIT

Try another like this

x = 0.77 or -0.43

Solve the equation 3x2 = x + 1 giving your answers to 2 decimal places.

What would you like to do now?

Page 156: INTERMEDIATE 2 – ADDITIONAL QUESTION BANK UNIT 3 : Further Trig Algebraic Operations Quadratic Functions EXIT.

Comments

Begin Solution

Question 2

Quadratics Menu

Back to Home

2. Write down values of a, b & c.

3x2 – x – 1 = 0

a = 3 b = (-1) c = (-1)

3. Evaluate b2 – 4ac .

b2 – 4ac = (-1)2 – (4 x 3 x -1)

= 1 – (-12)

= 13Continue Solution

Solve the equation

3x2 = x + 1

giving your answers

to 2 decimal places.

1. Rearrange in quadratic form.

3x2 = x + 1

Page 157: INTERMEDIATE 2 – ADDITIONAL QUESTION BANK UNIT 3 : Further Trig Algebraic Operations Quadratic Functions EXIT.

Comments

Begin Solution

Continue Solution

Question 2

Quadratics Menu

Back to Home

2. Write down values of a, b & c.

3x2 –x – 1 = 0

a = 3 b = (-1) c = (-1)

3. Now use quadratic formula.

x = -b ± (b2 – 4ac )

2a

x = 1 ± 13

24. Rewrite with brackets and now use

calculator.

= (1 + 13) 2 or (1 - 13) 2

Solve the equation

3x2 = x + 1

giving your answers

to 2 decimal places.

Page 158: INTERMEDIATE 2 – ADDITIONAL QUESTION BANK UNIT 3 : Further Trig Algebraic Operations Quadratic Functions EXIT.

Comments

Begin Solution

Question 2

Quadratics Menu

Back to Home

1. Write down values of a, b & c.

3x2 – x – 1 = 0

a = 3 b = (-1) c = (-1)

4. Rewrite with brackets and now use calculator.

= (1 + 13) 2 or (1 - 13) 2

5. Remember to round.

x = 0.77 or -0.43Try another like this

Solve the equation

3x2 = x + 1

giving your answers

to 2 decimal places.

= 0.767… or -0.434..

What would you like to do now?

Page 159: INTERMEDIATE 2 – ADDITIONAL QUESTION BANK UNIT 3 : Further Trig Algebraic Operations Quadratic Functions EXIT.

Comments

Quadratics Menu

Back to Home

Next Comment

For any question involving a quadratic in which you are asked to give your answer to a given number of decimal places

OR

a given number of significant figures:

THINK QUADRATIC FORMULA!!

2. Write down values of a, b & c.

3x2 – x – 1 = 0

a = 3 b = (-1) c = (-1)

3. Evaluate b2 – 4ac .

b2 – 4ac = (-1)2 – (4 x 3 x -1)

= 1 – (-12)

= 13

1. Rearrange in quadratic form.

3x2 = x + 1

Page 160: INTERMEDIATE 2 – ADDITIONAL QUESTION BANK UNIT 3 : Further Trig Algebraic Operations Quadratic Functions EXIT.

Comments

Quadratics Menu

Back to Home

Next Comment

You must put the quadratic into standard quadratic form (make equation equal zero) before attempting to solve.

2. Write down values of a, b & c.

3x2 – x – 1 = 0

a = 3 b = (-1) c = (-1)

3. Evaluate b2 – 4ac .

b2 – 4ac = (-1)2 – (4 x 3 x -1)

= 1 – (-12)

= 13

1. Rearrange in quadratic form.

3x2 = x + 1

Page 161: INTERMEDIATE 2 – ADDITIONAL QUESTION BANK UNIT 3 : Further Trig Algebraic Operations Quadratic Functions EXIT.

Comments

Quadratics Menu

Back to Home

Next Comment

ax2 + bx + c = 0

Refer to the Formula Sheet:

x = -b ± (b2 – 4ac )

2a

Quadratic Formula:

2. Write down values of a, b & c.

3x2 –x – 1 = 0

a = 3 b = (-1) c = (-1)

3. Now use quadratic formula.

x = -b ± (b2 – 4ac )

2a

x = 1 ± 13

24. Rewrite with brackets and now

use calculator.

= (1 + 13) 2 or (1 - 13) 2

Page 162: INTERMEDIATE 2 – ADDITIONAL QUESTION BANK UNIT 3 : Further Trig Algebraic Operations Quadratic Functions EXIT.

Comments

Quadratics Menu

Back to Home

Next Comment

Take care when allocating a

value to a, b and c.

ax2 + bx + c = 0

3x2 - 1x - 1 = 0

a =3b = (-1)c = (-1)

2. Write down values of a, b & c.

3x2 –x – 1 = 0

a = 3 b = (-1) c = (-1)

3. Now use quadratic formula.

x = -b ± (b2 – 4ac )

2a

x = 1 ± 13

24. Rewrite with brackets and now

use calculator.

= (1 + 13) 2 or (1 - 13) 2

Page 163: INTERMEDIATE 2 – ADDITIONAL QUESTION BANK UNIT 3 : Further Trig Algebraic Operations Quadratic Functions EXIT.

Note:

Finding first eases working.

Bracket all negative numbers. Watch out for double negative!

Comments

Quadratics Menu

Back to Home

b2 – 4ac

2. Write down values of a, b & c.

3x2 – x – 1 = 0

a = 3 b = (-1) c = (-1)

3. Evaluate b2 – 4ac .

b2 – 4ac = (-1)2 – (4 x 3 x -1)

= 1 – (-12)

= 13

1. Rearrange in quadratic form.

3x2 = x + 1

Try another like this

Page 164: INTERMEDIATE 2 – ADDITIONAL QUESTION BANK UNIT 3 : Further Trig Algebraic Operations Quadratic Functions EXIT.

Quadratic Functions: Question 2B

Go to full solution

Go to Comments

Go to Quadratics Menu

Reveal answer only

EXIT

Get hint

Solve the equation 2x(x + 2) = 2 - x

giving your answers to 2 significant figures.

What would you like to do now?

Page 165: INTERMEDIATE 2 – ADDITIONAL QUESTION BANK UNIT 3 : Further Trig Algebraic Operations Quadratic Functions EXIT.

Quadratic Functions: Question 2B

Go to full solution

Go to Comments

Go to Quadratics Menu

Reveal answer only

EXIT

Write down values of a,

b & c.

Evaluate b2 – 4ac .

Now use quadratic formula,

remembering that you should get

two answers.

Remember to round.

Solve the equation 2x(x + 2) = 2 - x

giving your answers to 2 significant figures.

Re-write quadratic: get rid of

brackets & make it equal to

zero before solving.

What would you like to do now?

Page 166: INTERMEDIATE 2 – ADDITIONAL QUESTION BANK UNIT 3 : Further Trig Algebraic Operations Quadratic Functions EXIT.

Quadratic Functions: Question 2B

Go to full solution

Go to Comments

Go to Quadratics Menu

EXIT

x = 0.35 or -2.9

Solve the equation 2x(x + 2) = 2 - x

giving your answers to 2 significant figures.

What would you like to do now?

Page 167: INTERMEDIATE 2 – ADDITIONAL QUESTION BANK UNIT 3 : Further Trig Algebraic Operations Quadratic Functions EXIT.

Comments

Begin Solution

Continue Solution

Question 2B

Quadratics Menu

Back to Home

Solve the equation

2x(x + 2) = 2 - x

giving your answers

to 2 significant figures.

2. Write down values of a, b & c.

2x2 + 5x – 2 = 0

a = 2 b = 5 c = (-2)

3. Evaluate b2 – 4ac .

b2 – 4ac = (5)2 – (4 x 2 x -2)

= 25 – (-16)

= 41

1. Rearrange in quadratic form.

2x(x + 2) = 2 – x 2x2 + 4x – 2 + x= 0

Page 168: INTERMEDIATE 2 – ADDITIONAL QUESTION BANK UNIT 3 : Further Trig Algebraic Operations Quadratic Functions EXIT.

Comments

Begin Solution

Continue Solution

Question 2B

Quadratics Menu

Back to Home

3. Now use quadratic formula.

x = -b ± (b2 – 4ac )

2a

x = -5 ± 41

24. Rewrite with brackets and now use

calculator.

= (-5 + 41) 2 or (-5 - 41) 2

Solve the equation

2x(x + 2) = 2 - x

giving your answers

to 2 significant figures.

2x2 + 5x – 2 = 0

a = 2 b = 5 c = (-2)

1. Rearrange in quadratic form.

Page 169: INTERMEDIATE 2 – ADDITIONAL QUESTION BANK UNIT 3 : Further Trig Algebraic Operations Quadratic Functions EXIT.

Comments

Begin Solution

Continue Solution

Question 2B

Quadratics Menu

Back to Home

4. Rewrite with brackets and now use calculator.

= (-5 + 41) 2 or (-5 - 41) 2

5. Remember to round.

x = 0.35 or -2.9

2x2 + 5x – 2 = 0

a = 2 b = 5 c = (-2)

1. Rearrange in quadratic form.

Solve the equation

2x(x + 2) = 2 - x

giving your answers

to 2 significant figures.

= 0.350… or -2.850..

What would you like to do now?

Page 170: INTERMEDIATE 2 – ADDITIONAL QUESTION BANK UNIT 3 : Further Trig Algebraic Operations Quadratic Functions EXIT.

Comments

Quadratics Menu

Back to Home

Next Comment

For any question involving a quadratic in which you are asked to give your answer to a given number of decimal places

OR

a given number of significant figures:

THINK QUADRATIC FORMULA!!

2. Write down values of a, b & c.

2x2 + 5x – 2 = 0

a = 2 b = 5 c = (-2)

3. Evaluate b2 – 4ac .

b2 – 4ac = (5)2 – (4 x 2 x -2)

= 25 – (-16)

= 41

1. Rearrange in quadratic form.

2x(x + 2) = 2 – x 2x2 + 4x – 2 + x= 0

Page 171: INTERMEDIATE 2 – ADDITIONAL QUESTION BANK UNIT 3 : Further Trig Algebraic Operations Quadratic Functions EXIT.

Comments

Quadratics Menu

Back to Home

Next Comment

2. Write down values of a, b & c.

2x2 + 5x – 2 = 0

a = 2 b = 5 c = (-2)

3. Evaluate b2 – 4ac .

b2 – 4ac = (5)2 – (4 x 2 x -2)

= 25 – (-16)

= 41

1. Rearrange in quadratic form.

2x(x + 2) = 2 – x 2x2 + 4x – 2 + x= 0

You must put the quadratic into standard quadratic form (make equation equal zero) before attempting to solve.

Page 172: INTERMEDIATE 2 – ADDITIONAL QUESTION BANK UNIT 3 : Further Trig Algebraic Operations Quadratic Functions EXIT.

Comments

Quadratics Menu

Back to Home

Next Comment

ax2 + bx + c = 0

Refer to the Formula Sheet:

x = -b ± (b2 – 4ac )

2a

Quadratic Formula:

3. Now use quadratic formula.

x = -b ± (b2 – 4ac )

2a

x = -5 ± 41

24. Rewrite with brackets and now

use calculator.

= (-5 + 41) 2 or (-5 - 41) 2

2x2 + 5x – 2 = 0

a = 2 b = 5 c = (-2)

1. Rearrange in quadratic form.

Page 173: INTERMEDIATE 2 – ADDITIONAL QUESTION BANK UNIT 3 : Further Trig Algebraic Operations Quadratic Functions EXIT.

Comments

Quadratics Menu

Back to Home

Next Comment

Take care when allocating a

value to a, b and c.

ax2 + bx + c = 0

2x2 + 5x - 2 = 0

a =2b = 5c = -2

3. Now use quadratic formula.

x = -b ± (b2 – 4ac )

2a

x = -5 ± 41

24. Rewrite with brackets and now

use calculator.

= (-5 + 41) 2 or (-5 - 41) 2

2x2 + 5x – 2 = 0

a = 2 b = 5 c = (-2)

1. Rearrange in quadratic form.

Page 174: INTERMEDIATE 2 – ADDITIONAL QUESTION BANK UNIT 3 : Further Trig Algebraic Operations Quadratic Functions EXIT.

Note:

Finding first eases working.

Bracket all negative numbers. Watch out for double negative!

Comments

Quadratics Menu

Back to Home

Next Comment

b2 – 4ac

2. Write down values of a, b & c.

2x2 + 5x – 2 = 0

a = 2 b = 5 c = (-2)

3. Evaluate b2 – 4ac .

b2 – 4ac = (5)2 – (4 x 2 x -2)

= 25 – (-16)

= 41

1. Rearrange in quadratic form.

2x(x + 2) = 2 – x 2x2 + 4x – 2 + x= 0

Page 175: INTERMEDIATE 2 – ADDITIONAL QUESTION BANK UNIT 3 : Further Trig Algebraic Operations Quadratic Functions EXIT.

Quadratic Functions: Question 3

Go to full solution

Go to Comments

Go to Quadratics MenuReveal answer

EXIT

Get hint

The diagram below shows an L-shaped plot of land with dimensions as given.

(x+4) m

x m

x m

(x+3) m

(a) Show that the total area is given by the expression x2 + 7x m2.

(b) Hence find the value of x when this area is 60 m2.

What would you like to do now?

Page 176: INTERMEDIATE 2 – ADDITIONAL QUESTION BANK UNIT 3 : Further Trig Algebraic Operations Quadratic Functions EXIT.

The diagram below shows an L-shaped plot of land with dimensions as given.

(x+4) m

x m

x m

(x+3) m

(a) Show that the total area is given by the expression x2 + 7x m2.

(b) Hence find the value of x when this area is 60 m2.

Quadratic Functions: Question 3

EXIT

Find the area of each rectangle..

Add these to find total area.

In (b) make expression from (a) =

60.

Make it equal to

zero, factorise &

solve.

Go to full solution

Go to Comments

Go to Quadratics Menu

Reveal answer

What would you like to do now?

Page 177: INTERMEDIATE 2 – ADDITIONAL QUESTION BANK UNIT 3 : Further Trig Algebraic Operations Quadratic Functions EXIT.

The diagram below shows an L-shaped plot of land with dimensions as given.

(x+4) m

x m

x m

(x+3) m

(a) Show that the total area is given by the expression x2 + 7x m2.

(b) Hence find the value of x when this area is 60 m2.

Quadratic Functions: Question 3

EXIT

Try another like this

x = 5m

Go to full solution

Go to Comments

Go to Quadratics Menu

What would you like to do now?

Page 178: INTERMEDIATE 2 – ADDITIONAL QUESTION BANK UNIT 3 : Further Trig Algebraic Operations Quadratic Functions EXIT.

Comments

Begin Solution

Question 3

Quadratics Menu

Back to Home

Continue Solution

(x+4) m

x m

x m

(x+3) m

1. Find the area of each rectangle separately and add together to get total area.

Area rectangle = x(x+4)

= x2 + 4x m2

(a)1

2

1

Length rectangle = (x + 3)2

= 3m

– x

= 3x m2Area rectangle 2

Hence total area = (x2 + 4x) + 3x

= x2 + 7x m2

(a) Show that the total area is given by the expression x2 + 7x m2.

3m

Page 179: INTERMEDIATE 2 – ADDITIONAL QUESTION BANK UNIT 3 : Further Trig Algebraic Operations Quadratic Functions EXIT.

Comments

Begin Solution

Question 3

Quadratics Menu

Back to Home

(x+4) m

x m

x m

(x+3) m

2. Use the expression for area given in part (a) and make it equal to 60.

If total area = 60 (b)

1

2

And total area = x2 + 7x

(b) Hence find the value of x

when this area is 60 m2.

then x2 + 7x = 60

x2 + 7x - 60 = 0

(x + 12)(x – 5) = 0

So (x + 12) = 0 or (x – 5) = 0

ie x = -12 or x = 5

Length must be 5m as negative

value not valid.

3. Make the equation equal to zero, factorise and solve.

Try another like this

What would you like to do now?

Page 180: INTERMEDIATE 2 – ADDITIONAL QUESTION BANK UNIT 3 : Further Trig Algebraic Operations Quadratic Functions EXIT.

Comments

Quadratics Menu

Back to Home

Next Comment

2. Use the expression for area given in part (a) and make it equal to 60.

If total area = 60 (b)

And total area = x2 + 7x then x2 + 7x = 60

x2 + 7x - 60 = 0

(x + 12)(x – 5) = 0

So (x + 12) = 0 or (x – 5) = 0

ie x = -12 or x = 5

Length must be 5m as negative

value not valid.

3. Make the equation equal to zero, factorise and solve.

The quadratic equation can also be solved by applyingthe quadratic formula:

ax2 + bx + c = 0

Refer to the Formula Sheet:

x = -b ± (b2 – 4ac )

2a

Quadratic Formula:

Finding first eases working.Bracket all negative numbers.

b2 – 4ac

Page 181: INTERMEDIATE 2 – ADDITIONAL QUESTION BANK UNIT 3 : Further Trig Algebraic Operations Quadratic Functions EXIT.

Comments

Quadratics Menu

Back to Home

Next Comment

2. Use the expression for area given in part (a) and make it equal to 60.

If total area = 60 (b)

And total area = x2 + 7x then x2 + 7x = 60

x2 + 7x - 60 = 0

(x + 12)(x – 5) = 0

So (x + 12) = 0 or (x – 5) = 0

ie x = -12 or x = 5

Length must be 5m as negative

value not valid.

3. Make the equation equal to zero, factorise and solve.

Take care when allocating a

value to a, b and c.

ax2 + bx + c = 0

1x2 + 7x - 60 = 0

a =1b = 7c = (-60)

Page 182: INTERMEDIATE 2 – ADDITIONAL QUESTION BANK UNIT 3 : Further Trig Algebraic Operations Quadratic Functions EXIT.

Comments

Quadratics Menu

Back to Home

2. Use the expression for area given in part (a) and make it equal to 60.

If total area = 60 (b)

And total area = x2 + 7x then x2 + 7x = 60

x2 + 7x - 60 = 0

(x + 12)(x – 5) = 0

So (x + 12) = 0 or (x – 5) = 0

ie x = -12 or x = 5

Length must be 5m as negative

value not valid.

3. Make the equation equal to zero, factorise and solve.

Make sure that you understand when a

negative answer may not be

correct in context given,

e.g. negative lengths or areas.

Try another like this

Page 183: INTERMEDIATE 2 – ADDITIONAL QUESTION BANK UNIT 3 : Further Trig Algebraic Operations Quadratic Functions EXIT.

Quadratic Functions: Question 3B

Go to full solution

Go to Comments

Go to Quadratics Menu

Reveal answer

EXIT

Get hint

The diagram below shows plans of the foundations for a house and garage.

12m

7m house

x m

? mgarage

Planning regulations state that the length of the garage should be 4m longer than its width and its floor area should be 25% of the floor area of the house.

(b) Hence find the value of x

to meet this requirement.

(a)Find an expression for the area of the garage in terms of x.

What would you like to do now?

Page 184: INTERMEDIATE 2 – ADDITIONAL QUESTION BANK UNIT 3 : Further Trig Algebraic Operations Quadratic Functions EXIT.

(b) Hence find the value of x

to meet this requirement.

(a)Find an expression for the area of the garage in terms of x.

Quadratic Functions: Question 3B

EXIT

Area = length x

width. Use information

to find length of garage.

Find the numerical value of the

required area for the garage.

Use expression from (a) to

form a quadratic equation.

Make it equal to

zero, factorise &

solve.

Go to full solution

Go to Comments

Go to Quadratics Menu

Reveal answer

The diagram below shows plans of the foundations for a house and garage.

12m

7m house

x m

? mgarage

Planning regulations state that the length of the garage should be 4m longer than its width and its floor area should be 25% of the floor area of the house.

What would you like to do now?

Page 185: INTERMEDIATE 2 – ADDITIONAL QUESTION BANK UNIT 3 : Further Trig Algebraic Operations Quadratic Functions EXIT.

The diagram below shows plans of the foundations for a house and garage.

12m

7m house

x m

? mgarage

Planning regulations state that the length of the garage should be 4m longer than its width and its floor area should be 25% of the floor area of the house.

(b) Hence find the value of x

to meet this requirement.

(a)Find an expression for the area of the garage in terms of x.

Quadratic Functions: Question 3B

EXIT Go to full solution

Go to Comments Go to Quadratics Menu

= x2 + 4x m2

x = 3What would you like to do now?

Page 186: INTERMEDIATE 2 – ADDITIONAL QUESTION BANK UNIT 3 : Further Trig Algebraic Operations Quadratic Functions EXIT.

Comments

Begin Solution

Question 3B

Quadratics Menu

Back to Home

Continue Solution

(a)

12m

7m house

x m

? m

garage

the length of the garage should

be 4m longer than its width and

its floor area should be 25%

of the floor area of the house.

Length of garage = (x+4) m

So area = x(x+4)

= x2 + 4x m2

1. Area = length x width. Width is x but still need to find length.

Page 187: INTERMEDIATE 2 – ADDITIONAL QUESTION BANK UNIT 3 : Further Trig Algebraic Operations Quadratic Functions EXIT.

Comments

Begin Solution

Question 3B

Quadratics Menu

Back to Home

Continue Solution

2. Area = length x width. Find the required area and use your answer to (a) to establish quadratic equation.

(b)

12m

7m house

x m

? m

garage

the length of the garage should

be 4m longer than its width and

its floor area should be 25%

of the floor area of the house.

Area house = 12 x 7 = 84m2

Area garage = 25% of 84m2

= 21m2

So x2 + 4x = 21

x2 + 4x - 21 = 0

(x + 7)(x – 3) = 0

(x + 7) = 0 or (x – 3) = 0

3. Make the equation equal to zero, factorise and solve.

Page 188: INTERMEDIATE 2 – ADDITIONAL QUESTION BANK UNIT 3 : Further Trig Algebraic Operations Quadratic Functions EXIT.

Comments

Begin Solution

Question 3B

Quadratics Menu

Back to Home

Continue Solution

(b)

12m

7m house

x m

? m

garage

the length of the garage should

be 4m longer than its width and

its floor area should be 25%

of the floor area of the house.

x2 + 4x - 21 = 0

(x + 7)(x – 3) = 0

(x + 7) = 0 or (x – 3) = 0

3. Make the equation equal to zero, factorise and solve.

ie. x = -7 or x = 3

Length must be 3m as

negative value not valid.

What would you like to do now?

Page 189: INTERMEDIATE 2 – ADDITIONAL QUESTION BANK UNIT 3 : Further Trig Algebraic Operations Quadratic Functions EXIT.

Comments

Quadratics Menu

Back to Home

Next Comment

The quadratic equation can also be solved by applyingthe quadratic formula:

ax2 + bx + c = 0

Refer to the Formula Sheet:

x = -b ± (b2 – 4ac )

2a

Quadratic Formula:

Finding first eases working.Bracket all negative numbers.

b2 – 4ac

2. Area = length x width. Find the required area and use your answer to (a) to establish quadratic equation.

Area house = 12 x 7 = 84m2

Area garage = 25% of 84m2

= 21m2

So x2 + 4x = 21

x2 + 4x - 21 = 0

(x + 7)(x – 3) = 0

(x + 7) = 0 or (x – 3) = 0

3. Make the equation equal to zero, factorise and solve.

Page 190: INTERMEDIATE 2 – ADDITIONAL QUESTION BANK UNIT 3 : Further Trig Algebraic Operations Quadratic Functions EXIT.

Comments

Quadratics Menu

Back to Home

Next Comment

Take care when allocating a

value to a, b and c.

ax2 + bx + c = 0

1x2 + 4x - 21 = 0

a =1b = 4c = (-21)

2. Area = length x width. Find the required area and use your answer to (a) to establish quadratic equation.

Area house = 12 x 7 = 84m2

Area garage = 25% of 84m2

= 21m2

So x2 + 4x = 21

x2 + 4x - 21 = 0

(x + 7)(x – 3) = 0

(x + 7) = 0 or (x – 3) = 0

3. Make the equation equal to zero, factorise and solve.

Page 191: INTERMEDIATE 2 – ADDITIONAL QUESTION BANK UNIT 3 : Further Trig Algebraic Operations Quadratic Functions EXIT.

Comments

Quadratics Menu

Back to Home

Next Comment

Make sure that you understand when a

negative answer may not be

correct in context given,

e.g. negative lengths or areas.

(b)

x2 + 4x - 21 = 0

(x + 7)(x – 3) = 0

(x + 7) = 0 or (x – 3) = 0

3. Make the equation equal to zero, factorise and solve.

ie. x = -7 or x = 3

Length must be 3m as

negative value not valid.

Page 192: INTERMEDIATE 2 – ADDITIONAL QUESTION BANK UNIT 3 : Further Trig Algebraic Operations Quadratic Functions EXIT.

(a) State the coordinates of E.

(b) The second parabola cuts the X-axis at C & D. D is the point (8,0), find the coordinates of C.

(c) Find the equation of the parabola with minimum turning point G.

Quadratic Functions: Question 4

Go to full solution

Go to CommentsReveal answer

EXIT Get hint

The blades in electric hair-clippers have grooves in the form of identical adjacent parabolas.

The second parabola has turning point E and equation y = (x – 5)2 – 9 .

X

Y

E F G

C D

Page 193: INTERMEDIATE 2 – ADDITIONAL QUESTION BANK UNIT 3 : Further Trig Algebraic Operations Quadratic Functions EXIT.

The second parabola has turning point E and equation y = (x – 5)2 – 9 .

(a) State the coordinates of E.

(b) The second parabola cuts the X-axis at C & D. D is the point (8,0), find the coordinates of C.

(c) Find the equation of the parabola with minimum turning point G.

In ‘completed square’, a

minimum value occurs when

bracket is zero.

Quadratic Functions: Question 4

EXIT

Use symmetry of a parabola to find other root if you know one.

Go to full solution

Go to Comments

Go to Quadratics Menu

Reveal answer

The blades in electric hair-clippers have grooves in the form of identical adjacent parabolas.

X

Y

E F G

C D

roots are equidistant from axis of symmetry.

use ‘completed square’ form to write equation of parabola.

What would you like to do now?

Page 194: INTERMEDIATE 2 – ADDITIONAL QUESTION BANK UNIT 3 : Further Trig Algebraic Operations Quadratic Functions EXIT.

(a) State the coordinates of E.

(b) The second parabola cuts the X-axis at C & D. D is the point (8,0), find the coordinates of C.

(c) Find the equation of the parabola with minimum turning point G.

The blades in electric hair-clippers have grooves in the form of identical adjacent parabolas.

The second parabola has turning point E and equation y = (x – 5)2 – 9 .

X

Y

E F G

C D

Quadratic Functions: Question 4

EXIT

Try another like this

Go to full solution

Go to Comments

Go to Quadratics Menu

E is the point (5,-9).

C is (2,0).

y = (x – 17)2 - 9

Page 195: INTERMEDIATE 2 – ADDITIONAL QUESTION BANK UNIT 3 : Further Trig Algebraic Operations Quadratic Functions EXIT.

Comments

Begin Solution

Question 4

Quadratics Menu

Back to Home

Continue Solution

1. In ‘completed square’, a minimum value occurs when bracket is zero.

X

Y

E F G

C D

The second parabola has

turning point E and equation

y = (x – 5)2 – 9 .

(a)

y will have a minimum value of

-9 when (x – 5)2 = 0.

ie when x = 5

so E is the point (5,-9).

Page 196: INTERMEDIATE 2 – ADDITIONAL QUESTION BANK UNIT 3 : Further Trig Algebraic Operations Quadratic Functions EXIT.

Comments

Begin Solution

Question 4

Quadratics Menu

Back to Home

Continue Solution

2. Use symmetry of a parabola to find other root if you know one.

X

Y

E F G

C D

E is the point (5,-9).

(b) D is the point (8,0),

find the coordinates of C.

(b) Horizontal dist from E to D

= horizontal dist from E to C = 3 units

D has x-coordinate 8

so x-coordinate of C is 6 units less ie C is (2,0).

Page 197: INTERMEDIATE 2 – ADDITIONAL QUESTION BANK UNIT 3 : Further Trig Algebraic Operations Quadratic Functions EXIT.

Comments

Begin Solution

Question 4

Quadratics Menu

Back to Home

3. Use symmetry of a parabola to find other turning points if you know one.X

Y

E F G

C D

E is the point (5,-9).

(c)Distance from E to G

is 12 units so G is (17,-9)

4. If you know turning point, use ‘completed square’ form to write equation of parabola.

Equation of this parabola

in same form as the second

ie y = (x – 17)2 - 9Try another like this

(c) Find the equation of the

parabola with minimum G.

What would you like to do now?

Page 198: INTERMEDIATE 2 – ADDITIONAL QUESTION BANK UNIT 3 : Further Trig Algebraic Operations Quadratic Functions EXIT.

Comments

Quadratics Menu

Back to Home

Next Comment

Learn the following Results:

When a quadratic equation is in

the form y = (x – a)2 + b

the parabola has a

minimum T.P. at (a,b).

When a quadratic equation is in the form y = b - (x – a)2 the parabola has a maximum T.P. at (a,b).

1. In ‘completed square’, a minimum value occurs when bracket is zero.

(a)

y will have a minimum value of

-9 when (x – 5)2 = 0.

ie when x = 5

so E is the point (5,-9).

Page 199: INTERMEDIATE 2 – ADDITIONAL QUESTION BANK UNIT 3 : Further Trig Algebraic Operations Quadratic Functions EXIT.

Comments

Quadratics Menu

Back to Home

Next Comment

Learn the following Results:

y = (x – 5)2 - 9

Minimum T.P. at P(5,-9)

e.g.

P

1. In ‘completed square’, a minimum value occurs when bracket is zero.

(a)

y will have a minimum value of

-9 when (x – 5)2 = 0.

ie when x = 5

so E is the point (5,-9).

Page 200: INTERMEDIATE 2 – ADDITIONAL QUESTION BANK UNIT 3 : Further Trig Algebraic Operations Quadratic Functions EXIT.

Comments

Quadratics Menu

Back to Home

Try another like this

Learn the following Results:

P

The roots are equidistant from theturning point (or axis of symmetry)in the horizontal direction.

2. Use symmetry of a parabola to find other root if you know one.

(b) Horizontal dist from E to D

= horizontal dist from E to C = 3 units

D has x-coordinate 8

so y-coordinate of C is 6 units less ie C is (2,0).

Page 201: INTERMEDIATE 2 – ADDITIONAL QUESTION BANK UNIT 3 : Further Trig Algebraic Operations Quadratic Functions EXIT.

Quadratic Functions: Question 4B

Go to full solution

Go to CommentsReveal answer

EXIT Get hint

The top of an ornate fence consists of a series of congruent adjacent parabolas.

The first parabola has turning point U and equation y = 4 – (x – 3)2 .

(a) State the coordinates of U.

(b) If T is the point (5,0) then find the coordinates of V, the maximum turning point on the second parabola.

(c) Find the equation of the parabola with maximum turning point W.

U V W

X

YT

Page 202: INTERMEDIATE 2 – ADDITIONAL QUESTION BANK UNIT 3 : Further Trig Algebraic Operations Quadratic Functions EXIT.

The top of an ornate fence consists of a series of adjacent parabolas.

The first parabola has turning point U and equation y = 4 – (x – 3)2 .

(a) State the coordinates of U.

(b) If T is the point (5,0) then find the coordinates of V, the maximum turning point on the second parabola.

(c) Find the equation of the parabola with maximum turning point W.

U V W

X

YT

Quadratic Functions: Question 4B

EXIT Go to full solution

Go to Comments

Go to Quadratics Menu

Reveal answer

In ‘completed square’, a

minimum value occurs when

bracket is zero.

Use symmetry of a parabola to find another tp if you know one.

use ‘completed square’ form to write equation of parabola.

What would you like to do now?

Page 203: INTERMEDIATE 2 – ADDITIONAL QUESTION BANK UNIT 3 : Further Trig Algebraic Operations Quadratic Functions EXIT.

(a) State the coordinates of U.

(b) If T is the point (5,0) then find the coordinates of V, the maximum turning point on the second parabola.

(c) Find the equation of the parabola with maximum turning point W.

Quadratic Functions: Question 4B

EXIT

Try another like this

Go to full solution

Go to Comments

Go to Quadratics Menu

U is the point (3,4).

V is (7,4).

y = 4 – (x – 11)2

The top of an ornate fence consists of a series of adjacent parabolas.

The first parabola has turning point U and equation y = 4 – (x – 3)2 .

U V W

X

YT

Page 204: INTERMEDIATE 2 – ADDITIONAL QUESTION BANK UNIT 3 : Further Trig Algebraic Operations Quadratic Functions EXIT.

Comments

Begin Solution

Question 4B

Quadratics Menu

Back to Home

Continue Solution

1. In ‘completed square’, a maximum value occurs when bracket is zero.

(a)

y will have a maximum value of

4 when (x – 3)2 = 0.

ie when x = 3

so U is the point (3,4).

The first parabola has turning

point U and equation

y = 4 – (x – 3)2 .

U V W

X

YT

Page 205: INTERMEDIATE 2 – ADDITIONAL QUESTION BANK UNIT 3 : Further Trig Algebraic Operations Quadratic Functions EXIT.

Comments

Begin Solution

Question 4B

Quadratics Menu

Back to Home

Continue Solution

2. Use symmetry of a parabola to find another turning point if you know one.

(b) T is the point (5,0),

find the coordinates of V.

(b) Horizontal dist from U to T

= horizontal dist from T to V = 2 units

T has x-coordinate 5

so x-coordinate of V is 2 units more

ie V is (7,4).

U V W

X

YT

U is the point (3,4).

For identical parabolae,maximum values same so y-coordinate = 4

Page 206: INTERMEDIATE 2 – ADDITIONAL QUESTION BANK UNIT 3 : Further Trig Algebraic Operations Quadratic Functions EXIT.

Comments

Begin Solution

Question 4B

Quadratics Menu

Back to Home

3. Use symmetry of a parabola to find other turning points if you know one.

(c)Distance from U to W

is 8 units so W is (11,4)

4. If you know turning point, use ‘completed square’ form to write equation of parabola.

Equation of this parabola

in same form as the first

ie y = 4 - (x – 11)2Try another like this

(c) Find the equation of the

parabola with maximum W.

U V W

X

YT

U is the point (3,4).

What would you like to do now?

Page 207: INTERMEDIATE 2 – ADDITIONAL QUESTION BANK UNIT 3 : Further Trig Algebraic Operations Quadratic Functions EXIT.

Comments

Quadratics Menu

Back to Home

Next Comment

Learn the following Results:

When a quadratic equation is in

the form y = (x – a)2 + b

the parabola has a

minimum T.P. at (a,b).

When a quadratic equation is in the form y = b - (x – a)2 the parabola has a maximum T.P. at (a,b).

1. In ‘completed square’, a maximum value occurs when bracket is zero.

(a)

y will have a maximum value of

4 when (x – 3)2 = 0.

ie when x = 3

so U is the point (3,4).

Page 208: INTERMEDIATE 2 – ADDITIONAL QUESTION BANK UNIT 3 : Further Trig Algebraic Operations Quadratic Functions EXIT.

Comments

Quadratics Menu

Back to Home

Learn the following Results:

y = 4 – (x – 3)2

Maximum T.P. at P(3,4)

e.g.

P

1. In ‘completed square’, a maximum value occurs when bracket is zero.

(a)

y will have a maximum value of

4 when (x – 3)2 = 0.

ie when x = 3

so U is the point (3,4).

Try another like this

Page 209: INTERMEDIATE 2 – ADDITIONAL QUESTION BANK UNIT 3 : Further Trig Algebraic Operations Quadratic Functions EXIT.

(a) State the coordinates of G.

(b) Find the coordinates of H and J.

(c) Find the equation of the parabola with minimum turning point at F.

Quadratic Functions: Question 4C

Go to full solution

Go to CommentsReveal answer

EXIT Get hint

The diagram below shows some identical adjacent parabolas.

The last parabola has a minimum turning point at G and cuts the x-axis at H and J.

Its equation is y = (x – 12)2 – 4 .

X

Y

F G

H J

Page 210: INTERMEDIATE 2 – ADDITIONAL QUESTION BANK UNIT 3 : Further Trig Algebraic Operations Quadratic Functions EXIT.

(a) State the coordinates of G.

(b) Find the coordinates of H and J.

(c) Find the equation of the parabola with minimum turning point at F.

The last parabola has a minimum turning point at G and cuts the x-axis at H and J.

Its equation is y = (x – 12)2 – 4 . In ‘completed

square’, a minimum value

occurs when bracket is zero.

Quadratic Functions: Question 4C

EXIT

If not given a root you must solve the quadratic.

Go to full solution

Go to Comments

Go to Quadratics Menu

Reveal answer

use ‘completed square’ form to write equation of parabola.

The diagram below shows some identical adjacent parabolas.

X

Y

F G

H J

What would you like to do now?

Page 211: INTERMEDIATE 2 – ADDITIONAL QUESTION BANK UNIT 3 : Further Trig Algebraic Operations Quadratic Functions EXIT.

(a) State the coordinates of G.

(b) Find the coordinates of H and J.

(c) Find the equation of the parabola with minimum turning point at F.

Quadratic Functions: Question 4C

EXIT Go to full solution

Go to Comments

Go to Quadratics Menu

G is the point (12,-4).

y = (x – 8)2 - 4

The diagram below shows some identical adjacent parabolas.

The last parabola has a minimum turning point at G and cuts the x-axis at H and J.

Its equation is y = (x – 12)2 – 4 .

X

Y

F G

H J

H is (10,0) and J is (14,0)

What would you like to do now?

Page 212: INTERMEDIATE 2 – ADDITIONAL QUESTION BANK UNIT 3 : Further Trig Algebraic Operations Quadratic Functions EXIT.

Comments

Begin Solution

Question 4C

Quadratics Menu

Back to Home

Continue Solution

1. In ‘completed square’, a minimum value occurs when bracket is zero.

(a)

y will have a minimum value of

-4 when (x – 12)2 = 0.

ie when x = 12

so G is the point (12,-4).

X

Y

F G

H J

Its equation is y = (x – 12)2 – 4 .

Page 213: INTERMEDIATE 2 – ADDITIONAL QUESTION BANK UNIT 3 : Further Trig Algebraic Operations Quadratic Functions EXIT.

Comments

Begin Solution

Question 4C

Quadratics Menu

Back to Home

Continue Solution

2. If you are not given one of the roots you must solve quadratic to find roots.

G is the point (12,-4).

X

Y

F G

H J

(b) Find the coordinates of

H and J.

(b) At H & J

(x – 12)2 – 4 = 0

so (x – 12)2 = 4

ie x – 12 = -2 or 2

ie x = 10 or 14

H is (10,0) and J is (14,0)

From diagram:

y = (x – 12)2 – 4 .

Page 214: INTERMEDIATE 2 – ADDITIONAL QUESTION BANK UNIT 3 : Further Trig Algebraic Operations Quadratic Functions EXIT.

X

Y

F G

H J

Comments

Begin Solution

Question 4C

Quadratics Menu

Back to Home

3. Use symmetry of a parabola to find other turning points if you know one.

(c)Distance from G to H

is 2 units so F is 4 units horizontally from G.

4. If you know turning point, use ‘completed square’ form to write equation of parabola.

Equation of this parabola

in same form as the last

(c) Find the equation of the

parabola with minimum F.

G is the point (12,-4).

10 14

So F is the point (8,-4).

ie y = (x – 8)2 - 4

Continue Solution

What would you like to do now?

Page 215: INTERMEDIATE 2 – ADDITIONAL QUESTION BANK UNIT 3 : Further Trig Algebraic Operations Quadratic Functions EXIT.

Comments

Quadratics Menu

Back to Home

Next Comment

Learn the following Results:

When a quadratic equation is in

the form y = (x – a)2 + b

the parabola has a

minimum T.P. at (a,b).

When a quadratic equation is in the form y = b - (x – a)2 the parabola has a maximum T.P. at (a,b).

1. In ‘completed square’, a minimum value occurs when bracket is zero.

(a)

y will have a minimum value of

-4 when (x – 12)2 = 0.

ie when x = 12

so G is the point (12,-4).

Page 216: INTERMEDIATE 2 – ADDITIONAL QUESTION BANK UNIT 3 : Further Trig Algebraic Operations Quadratic Functions EXIT.

Comments

Quadratics Menu

Back to Home

Next Comment

Learn the following Results:

y = (x – 12)2 – 4

Minimum T.P. at P(12,-4)

e.g.

P

1. In ‘completed square’, a minimum value occurs when bracket is zero.

(a)

y will have a minimum value of

-4 when (x – 12)2 = 0.

ie when x = 12

so G is the point (12,-4).

End of Unit 3


Recommended